Class Xi Study Material 2-23 24 (KVS, Jabalpur)

Download as pdf or txt
Download as pdf or txt
You are on page 1of 129

केंद्रीय विद्यालय संगठन

Kendriya Vidyalaya Sangathan

सत्र/ Session 2023-24


अध्ययन सामग्री गणित
Study Material Maths
कक्षा - ग्यारहिी
Class - XI

Kendriya Vidyalaya Sangathan (Jabalpur Region)


Regional Office, Jabalpur
Behind Science College,
South Civil Lines, Jabalpur (M.P.)- 482011
Website - https://rojabalpur.kvs.gov.in
केंद्रीय विद्यालय संगठन
Kendriya Vidyalaya Sangathan

हमारे संरक्षक
OUR PATRONS

श्री सोममत श्रीिास्ति


उपायक्
ु त, के.वि.स. क्षेत्रीय कायाालय जबलपरु

श्री हीरालाल श्रीमती सरोज डबास श्रीमती ककरण शमाा


सहायक आयुक्त सहायक आयुक्त सहायक आयुक्त

केंद्रीय विद्यालय संगठन क्षेत्रीय कायाालय जबलपुर


JABALPUR REGION

अध्ययन सामग्री/STUDY MATERIAL


सत्र/ SESSION: 2023-24
कक्षा/CLASS- ELEVENTH(XI)
गणित/MATHEMATICS
णवषय कोड/Subject Code - 041

DIRECTED BY :-
MR. PANKAJ KUMAR JAIN
PRINCIPAL, KV BALAGHAT

i
DIRECTOR’S MESSAGE

It is with profound delight and utmost pride that I announce the publication of our
study material for class XI (MATHEMATICS) for the session 2023-24. It’s my
firm belief that access to quality education should know no boundaries,
transcending social and economic constraints. Our collective vision is to
empower all students with the tools for success and intellectual growth.

With their steadfast dedication PGT-MATHEMATICS of Jabalpur Region of


Kendriya Vidyalaya Sangathan have invested their knowledge, expertise,
and passion into meticulously crafting these study materials to complement the
classroom learning experience of the students. These materials serve as
invaluable aids for self-study since they are comprehensive, well-structured,
and presented in a manner that is easy to comprehend.

It is with pleasure that I place on record my commendation for the commitment


and dedication of the team of all PGT Mathematics of Kendriya Vidyalaya
Sangathan Jabalpur Region.

Wishing you all the very best in your academic journey!

PANKAJ KUMAR JAIN

PRINCIPAL
KENDRIYA VIDYALAY
BALAGHAT

ii
CONTENT DEVELOPMENT TEAM OF PGT MATHEMATICS
BOOKLATE FOR CLASS - XI , SUBJECT :- MATHEMATICS (041)

S.NO. NAME OF CHAPTER PGT (MATHEMATICS) KENDRIYA VIDYALAYA

1 1. Sets. MR. DURAG SINGH DANGI AMARKANTAK

MR. VIVEK CHANDAMETTA


2 2. Relations & Functions
VISHWAKARMA BARKUHI (WCL)

3 3.Trigonometric Functions MRS. VANITA BAHEY CHHINDWARA NO.II

4. Complex Numbers and MR. DEVENDRA KUMAR


4 CHHINDWARA NO.II
Quadratic Equations SINGH

5 5. Linear Inequalities MR. DIGVIJAY SINGH DINDORI

6. Permutations and
6 MR R K TRIPATHI JAYANT COLLIERY
Combinations

MR. RAKESH KUMAR


7 7.Binomial Theorem PATKAR JABALPUR GCF NO.I

8 8. Sequence and Series Mrs. SARITA SINGH SAGAR NO.I (CANT.)

9 9. Straight Lines Mrs. REKHA BARETHIYA SAGAR NO.I (CANT.)

10 10. Conic Sections Mrs. Rashmi Soni KATNI NO.II (RLY)

11. Introduction to Three- MR JITENDRA KUMAR


11 KATNI NO.I (OF)
dimensional Geometry TIWARI

MR. RAJENDRA KUMAR


12 12. Limits and Derivatives NOWROZABAD SECL
BARMAN

13 13.Statistics MR. Neeraj chaurasia PANNA

14 14.Probability MR. Anil Kumar Tamrakar REWA NO.II

REVIEW AND
15 MR. RITESH AGRAWAL BALAGHAT
COMPILATION
INDEX

S.NO
NAME OF CHAPTER PAGE NUMBER
.

1 1. Sets. 1-4

2 2. Relations & Functions 5-11

3 3.Trigonometric Functions 12-19

4. Complex Numbers and Quadratic


4 20-27
Equations

5 5. Linear Inequalities 28-31

6 6. Permutations and Combinations 32-40

7 7.Binomial Theorem 41-49

8 8. Sequence and Series 50-56

9 9. Straight Lines 57-63

10 10. Conic Sections 64-73

11. Introduction to Three-dimensional


11 74-76
Geometry

12 12. Limits and Derivatives 77-83

13 13.Statistics 84-96

14 14.Probability 97-101
1

KENDRIYA VIDYALAYA SANGATHAN JABALPUR


STUDY MATERIAL FOR STUDENTS: 2023-24
CLASS XI
Subject: Mathematics
(Chapter -1: Sets)
Points to Remember:

⮚ A set is a well-defined collection of objects.


⮚ A set which does not contain any element is called empty set.
⮚ A set which consists of a definite number of elements is called finite set, otherwise, the set is
called infinite set.
⮚ Two sets A and B are said to be equal if they have exactly the same elements.
⮚ A set A is said to be subset of a set B, if every element of A is also an element of B. Intervals are
subsets of R.
⮚ The union of two sets A and B is the set of all those elements which are either in A or in B.
⮚ The intersection of two sets A and B is the set of all elements which are common. The difference
of two sets A and B in this order is the set of elements which belong to A but not to B.
⮚ The complement of a subset A of universal set U is the set of all elements of U which are not the
elements of A.
⮚ For any two sets A and B, (A ∪ B)′ = A′ ∩ B′ and ( A ∩ B )′ = A′ ∪ B′
⮚ The collection of all subsets of a set A is called the power set of A. It is denoted by P(A). In P(A), every
element is a set.

In general, if A is a set with n(A) = m, then it can be shown that n [ P(A)] = 2n
⮚ In general, if A and B are finite sets, then n ( A ∪ B ) = n ( A ) + n ( B ) – n ( A ∩ B )
⮚ If n ( A ∩ B )= ⱷ then n ( A ∪ B ) = n ( A ) + n ( B )

Some Properties of Complement Sets


⮚ Complement laws: (i) A ∪ A′ = U (ii) A ∩ A′ = φ
⮚ De Morgan’s law: (i) (A ∪ B)´ = A′ ∩ B′ (ii) (A ∩ B )′ = A′ ∪ B′
⮚ Law of double complementation: (A′ )′ = A
⮚ Laws of empty set and universal set φ′ = U and U′ = φ.
,,,,,,,,,,,,,,,,,,,,,,,,,,,,,,,,,,,,,,,,,,,,,,,,,,,,,,,,,,,,,,,,,,,,,,,,,,,,,,,,,,,,,,,,,,,,,,,,,,,,,,,,,,,,,,,,,,,,,,,,,,,,,,,,,,,,,,,,,,,,,,,,,,,
MCQS

Q.1: If the sets A and B are given by A = {1, 2, 3, 4}, B = {2, 4, 6, 8, 10} and the universal set
U = {1, 2, 3, 4, 5, 6, 7, 8, 9, 10}, then:
(a) (A 𝖴 B)' = {5, 7, 9} (b) (A ∩ B)'= {1, 3, 5, 6, 7}
(c) (A ∩ B)' = {1, 3, 5, 6, 7, 8} (d) None of these
Q.2: If A = {1, 2, 3, 4}, B = {2, 3, 5, 6} and C = {3, 4, 6, 7}, then
(a) A – (B ∩ C) = {1, 3, 4} (b) A – (B ∩ C) = {1, 2, 4}
(c) A – (B 𝖴 C) = {2, 3} (d) A – (B 𝖴 C) = {Ø}
Q.3: The number of the proper subset of {a, b, c} is:
(a) 3 (b) 8 (c) 6 (d) 7
Q.4: Which one is different from the others?
(a) empty set (b) void set (c) zero set (d) null set
Q.5: Which of the following collections is a set ?
(a) The collection of all the days of a week (b) A collection of 11 best hockey player of India.
(c) The collection of all rich person of Delhi (d) A collection of most dangerous animals of India.
Q.6: If A 𝖴 B = Ø then n(A 𝖴 B) is equal to:
(a) n(A) + n(B) - n(A∩B) (b) n(A) - n(B) + n(A∩B)
2

(c) n(A) + n(B) + n(A∩B) (d) n(A) - n(B) - n(A∩B)


Q.7: The number of subsets of a set containing n elements is:
(a) n (b) 2𝑛- 1 (c ) 𝑛2 (d) 2𝑛
Q.8: Set A = {0, 7, 26, 63} in set-builder form is:
(a) {x: x ∈ N, x = n3 – 1 and n ≤ 7} (b) {x: x ∈ N, x = n2 – 1 and n ≤ 5}
(c) {x: x ∈ N, x = n2 – 1 and n < 4} (d) {x: x ∈ N, x = n3 – 1 and n ≤ 4}
Q.9: If B = {Ø} then:
(a) B is an empty set (b) B is a finite set
(c) B is an infinite set (d) B is not a set
Q.10: If n(A) = 2 then n[P(P(A))] is:
(a) 8 (b) 4 (c) 12 (d) 16
,,,,,,,,,,,,,,,,,,,,,,,,,,,,,,,,,,,,,,,,,,,,,,,,,,,,,,,,,,,,,,,,,,,,,,,,,,,,,,,,,,,,,,,,,,,,,,,,,,,,,,,,,,,,,,,,,,,,,,,,,,,,,,,,,,,,,,,,,,,,,,,,,,,,,,,,,,

THREE LEVELS OF GRADED QUESTIONS


Level –I
( Very Short Answer Type Questions- 2 Marks)

Q.1: Write down the set {5, 25, 125, 625} in the set builder form
Q.2: If n(A) = 2 then find n[P(P(A))]
Q.3: Write down {x: x ∈ R, - 4 < x ≤ 6} as interval.
Q.4: Write the set A ={x: x ∈ R, 2x + 11= 15} in the roaster form.

Q.5: If A and B are two sets such that A ⊂ B, then what is A 𝖴 B?


Q.6: If S = {x: x is a positive multiple of 3 less than 100} and
P = {x: x is a prime number less than 20}.Then find n(S) + n(P)
Q.7: Two finite sets have m and n elements. The total number of subsets of the first set is 56 more
than the total number of subsets of the second set. Find the values of m and n.
Q.8: Let S = the set of all triangles, P = the set of all isosceles triangles, Q = the set of all equilateral
triangles, R = the set of all right-angled triangles. What do the sets P ∩ Q and R – P represents
respectively ?
Q.9: Let V = {a, e, i, o, u}, V – B = {e, o} and B – V = {k}. Find the set B.
Q.10: If A = {a, {b}}, then find P(A)
……………………………………………………………………………………………………………………………………………………………………………………
Level –II
(Short Answer Type Questions- 3 Marks)

Q.1: Write down all the subsets of the set {1, 2, 3}


Q.2: Draw appropriate venn diagram for each of the following:
(a) (A 𝖴 B)’ (b) A – B
Q.3: Prove that A= B if P(A) = P(B)
Q.4: A survey shows that 63%of Indians like cheese where as 76% like apples. If x % of Indians like
cheese and apples. Find the value of x.
Q.5: Using properties of sets prove that (𝐴 𝖴 𝐵)′ = 𝐴′ ∩ 𝐵′ .
Q.6: Using Venn Diagram prove that: 𝐴 ∩ (𝐵 − 𝐶) = (𝐴 ∩ 𝐵) − (𝐴 ∩ 𝐶).
Q.7: For all sets A, B and C, if A ⊂B, then A 𝖴 C ⊂ B 𝖴 C
…………………………………………………………………………………………….
Level -III
(Long Answer Type Questions- 5 Marks)

Q.1: If A = {3, 5, 7, 9, 11}, B = {7, 9, 11, 13} , C= {11, 13, 15} , D = {15, 17}, Find
(a) (A 𝖴 B) ∩ C (b) C - D (c) (A ∩ B) ∩ (B 𝖴 C) (d) B ∩ D
Q.2: In a survey of 100 students the number of students studying the various languages were found
to be: English only18, English but not Hindi 23, English and Sanskrit 8, English 26, Sanskrit 48,
3

Sanskrit and Hindi 8, no language 24.Find


(i) How many students were studying Hindi?
(ii) How many students were studying English and Hindi?
(iii) How many students were studying Sanskrit only?
Q.3: In a survey of 25 students of a school it was found that 15 study Mathematics, 12 study Physics,
11 study Chemistry, 9 study both Mathematics & Physics, 4 study both Physics & Chemistry, 5
study both Chemistry and Mathematics and 3 students all of the above three subjects. Find the
number of students who study:
(a) Only Mathematics
(b) At least one of the three subjects.
Q.4: In a group of students ,100 students know Hindi, 50 know English and 25 know both . Each of the
students know either Hindi or English. How many students are there in the group .
Q.5: If n(A - B) = 18, n(A 𝖴 B) = 70 and n(A ∩ B) = 25, then find n(B).
Q.6: If L = {1, 2, 3, 4}, M = {3, 4, 5, 6} and N = {1, 3, 5}, then verify that
L − (M 𝖴N) = (L − M) ∩ (L − N).
Q.7: If A, B and C be sets. Then, show that A∩ (B 𝖴 C) = (A ∩ B) 𝖴 (A ∩ C).

,,,,,,,,,,,,,,,,,,,,,,,,,,,,,,,,,,,,,,,,,,,,,,,,,,,,,,,,,,,,,,,,,,,,,,,,,,,,,,,,,,,,,,,,,,,,,,,,,,,,,,,,,,,,,,,,,,,,,,,,,,,,,,,,,,,,,,,,,,,,,,,,,,,,,,,,,,,,,,,,,,,,,,,,,,,,,,,,,,,,,,
ASSERTION & REASON TYPE QUESTIONS
Directions : Each of these questions contains two statements, Assertion and Reason. Each of these
questions also has four alternative choices, only one of which is the correct answer. You have to select one
of the codes (a), (b), (c) and (d) given below.
(a) Assertion is correct, reason is correct; reason is a correct explanation for assertion.
(b) Assertion is correct, reason is correct; reason is not a correct explanation for assertion
(c) Assertion is correct, reason is incorrect
(d) Assertion is incorrect, reason is correct.
Q.1: Assertion : The number of non-empty subsets of the set {a, b, c, d} are 15.
Reason : Number of non-empty subsets of a set having n elements are 2n – 1.
Q.2: Suppose A, B and C are three arbitrary sets and U is a universal set.
Assertion : If B = U – A, then n(B) = n(U) – n(A).
Reason : If C = A – B, then n(C) = n(A) – n(B).
Q.3: Assertion : Let A = {1, {2, 3}}, then P(A) = {{1}, {2, 3}, Ø, {1, {2, 3}}}.
Reason : Power set is set of all subsets of A.
Q.4: Assertion : The subsets of the set {1, {2}} are { }, {1}, {{2}} and {1, {2}}.
Reason : The total number of proper subsets of a set containing n elements is 2n – 1.
Q.5: Assertion : For any two sets A and B, A – B ⊂ B’
Reason : If A be any set, then A ∩ A’ = Ø

,,,,,,,,,,,,,,,,,,,,,,,,,,,,,,,,,,,,,,,,,,,,,,,,,,,,,,,,,,,,,,,,,,,,,,,,,,,,,,,,,,,,,,,,,,,,,,,,,,,,,,,,,,,,,,,,,,,,,,,,,,,,,,,,,,,,,,,,,,,,,,,,,,,,,,,,,,,,,,,,,,,,,,,,,,,,,,

CRITICALTHINKING TYPE QUESTIONS

CASE BASED QUESTION-1:

Venn diagrams were invented by a logician John Venn as a way of picturing relationships between
different groups of things. These diagrams, also called Set diagrams or Logic diagrams,
are widely used in mathematics, statistics, logic, teaching, linguistics, computer science and
business
In the following diagram, triangle shows children, circle shows rural population, rectangle shows
school going population & square shows boys.
4

Based on the information stated above answer the below given questions-:
(i) The rural boys not going to school are denoted by which number?
(a)1 (b) 2 (c)1,2 (d) 2,8
(ii) The children from rural population not going to school are denoted by which number?
(a)1 (b) 2 (c) 6 (d) 2,6
(iii) What is represented by number 4?
(a) Children who are not from rural population (b) Children who are boys
(c) School going boys (d) School going boys who are not from rural population
(iv) School going boys from village are denoted by which number?
(a) 3 (b) 3,5 (c) 3,4 (d) 3, 4, 5,7
(v) Number of children who are not from rural population?
(a) 8 (b) 2 (c) 7 (d) 9

CASE BASED QUESTION-2:


In D.A.V School, Bahadurgarh, a survey was done on 400 students it was found that 100 like to
take apple juice , 150 like to take orange juice and 75 like both apple as well as orange juice.

Based on above information, answer the following questions:


(i) Number of students who like either of the drink :
(a) 400 (b) 175 (c) 250 (d) 325
(ii) Number of students who likes neither apple juice nor orange juice :
(a) 225 (b) 325 (c) 75 (d) 25
(iii) Number of students who likes only apple juice:
(a) 125 (b) 75 (c) 100 (d) 25
(iv) Number of students who likes only orange juice :
(a) 75 (b) 25 (c) 100 (d) 125
(v) Which information we get from the given data :
(a) n (A U B) =n(A) + n (B) (b) n( A U B ) < n (A Ո B)
(c) n(A U B) < n (U) (d) n ( A Ո B) = n(A) + n (B)
5

(Chapter -2: RELATION AND FUNCTION)


6

2 MARKS
7

Q10.

3 MARKS

Q.1

Q 5.

Q 6.
8

Q 7.

Q 10.

Q 11.

Q 12.

Q 13.

5 MARKS
9

, f (40) , f (-40) & f(35)


10
11
12

(Chapter -3: TRIGONOMETRIC FUNCTIONS)


MCQ
1. The value of tan 1° tan 2° tan 3° ... tan 89° is:
(a) 0 (b) 1 (c) 1⁄2 (d) Not defined
2. If α + β = π/4, then the value of (1 + tan α) (1 + tan β) is :
(a) 1 (b) 2 (c) – 2 (d) Not defined
Q.NO.3 The greatest and least values of sinx , cosx are respectively
𝟏 −𝟏 𝟏 −𝟏
a. 1, -1 b.𝟐 , 𝟐 c. 𝟒 , d.2,-2
𝟒

𝝅 𝟕𝝅 𝝅
Q.NO.4 Value of 2sin2𝟔 + cosec2 𝟔 +cos2 𝟑 is
𝟏 −𝟏 𝟑
a. 𝟐 b. 𝟐 c. 1 d.𝟐

Q.NO.5 The conversion of 40°20’ into radians is:


𝟏𝟓𝝅 𝟏𝟐𝟏𝝅
a. 𝝅 radians b. c. 𝟓𝟒𝟎 d. None of these
𝟗

ASSERTION-REASON BASED QUESTIONS


In the following questions, a statement of Assertion (A) is followed by a statement of Reason (R).
Choose the correct answer out of the following choices.
(a) Both (A) and (R) are true and (R) is the correct explanation of (A).
(b) Both (A) and (R) are true but (R) is not the correct explanation of (A).
(c) (A) is true but (R) is false.
(d) (A) is false but (R) is true.
Q.NO.1 ASSERTION (A):The ratio of the radii of two circles at the centres of which two equal arcs
substend angles of 30° and 70° is 21 : 10.
Reason(R):Number of radians in an angle subtended at the centre of a circle by an arc is equal to the
ratio of the length of the arc to the radius of the circle.
𝒄𝒐𝒔(𝝅+𝒙).𝒄𝒐𝒔(−𝒙)
Q.NO.2 ASSERTION (A): 𝝅 = cot2x
𝒔𝒊𝒏(𝝅−𝒙)(𝒄𝒐𝒔 +𝒙)
𝟐

Reason(R):cos(𝝅 + 𝜽) = -cos𝜽, cos(-𝜽) = cos 𝜽, sin(𝝅 − 𝜽) = sin𝜽 , sin(-𝜽) = sin 𝜽


2 MARKS QUESTIONS
Q.NO.1 Find the radius of the circle in which a central angle of 60° intercepts an arc of length 37.4 cm
(use π = 22/7).

Q.NO.2 Show that cos 60° + cos 120° + cos 240° - sin 300° = 0
𝟏+𝒔𝒊𝒏𝟐𝒙−𝒄𝒐𝒔𝟐𝒙
Q.NO.3 Show that 𝟏+𝒔𝒊𝒏𝟐𝒙+𝒄𝒐𝒔𝟐𝒙 = tanx
13

√𝟏−𝒔𝒊𝒏𝒙 √𝟏+𝒔𝒊𝒏𝒙
Q.NO.4 If x lies in second quadrant, then show that + = 2secx
√𝟏+𝒔𝒊𝒏𝒙 √𝟏−𝒔𝒊𝒏𝒙
𝒙−𝒚
Q.NO.5 Show that (cos x – cos y)2 + ( sin x – sin y)2 = 4 sin2( )
𝟐

3 MARKS QUESTIONS
𝜽 𝟗𝜽 𝟓𝜽
Q.NO.1 Show that cos 2𝜽.cos𝟐 - cos 3𝜽 . cos = sin 5𝜽 .sin 𝟐
𝟐

Q.NO.2 Prove that sin2A = cos2(A-B)+cos2B – 2cos(A-B)cosA.cosB


𝟑 −𝟏𝟐
Q.NO.3 If sin A = 𝟓 , cos B = , where A and B both lie in second quadrant , find the value of
𝟏𝟑
sin(A+B)
Q.NO.4 Prove that tan8x – tan6x – tan2x = tan9x.tan6x.tan2x
𝐬𝐢𝐧 𝑨+𝐬𝐢𝐧 𝟑𝑨+𝒔𝒊𝒏𝟓𝑨+𝒔𝒊𝒏𝟕𝑨
Q.NO.5 Prove that 𝐜𝐨𝐬 𝑨+𝒄𝒐𝒔𝟑𝑨+𝒄𝒐𝒔𝟓𝑨+𝒄𝒐𝒔𝟕𝑨 = tan 4A

5 MARKS QUESTIONS
−𝟓 𝒙 𝒙 𝒙
Q.NO.1 If tanx = 𝟏𝟐 and x lies in 2nd quadrant , find the value of sin𝟐 , cos𝟐 , tan𝟐
𝝅 𝝅 𝟑
Q.NO.2 Prove that cos2x + cos2(x +𝟑 )+cos2(x - ) =𝟐
𝟑

Q.NO.3 Prove that cos 6x =32cos6x – 48cos4x +18cos2x -1


CASE BASED QUESTIONS
Q.NO.1 Rajiv constructs two right angled triangles in the fourth quadrant in such a way that the
𝟒 𝟏𝟐 𝟑𝝅
measure of triangle gives cos A = 𝟓 and cos B =𝟏𝟑 where 𝟐 < 𝑨 and B< 𝟐𝝅.

Based on the above information ,answer the following questions


1) Find the value of cos(A+B)
2) Find the value of sin(A-B)
3) Find the value of tan(A+B)
𝝅
Q.NO.2 In a class test of class XI , a teacher asked to students to consider A + B = 𝟒 , where A and B are
acute angles.
Based on the above information ,answer the following questions
1) Find the value of (1+tan A).(1- tanB)
2) Find the value of (cot B - 1).(cot A - 1)
3) Find the value of sin(A+B)-cos(A+B)+tan(A+B)

MARKING SCHEME

Q.NO.1 Correct option: (b) 1

Solution: tan 1° tan 2° tan 3° ... tan 89°

= [tan 1° tan 2° ... tan 44°] tan 45°[tan (90° – 44°) tan (90° – 43°)... tan (90° – 1°)]
14

= [tan 1° tan 2° ... tan 44°] [cot 44° cot 43°....... cot 1°] × [tan 45°]

= [(tan 1° × cot 1°) (tan 2° × cot 2°) .....(tan 44° × cot 44°)] × [tan 45°]

We know that, tan A × cot A =1 and tan 45° = 1

Hence, the equation becomes as;

= 1 × 1 × 1 × 1 × ...× 1= 1 {As 1n = 1}

Q.NO.2 Correct option: (b) 2

Solution: Given, α + β = π/4

Taking “tan” on both sides,

tan(α + β) = tan π/4

We know that, tan(A + B) = (tan A + tan B)/(1 – tan A tan B)

and tan π/4 = 1.

So, (tan α + tan β)/(1 – tan α tan β) = 1

tan α + tan β = 1 – tan α tan β

tan α + tan β + tan α tan β = 1....(i)

(1 + tan α)(1 + tan β) = 1 + tan α + tan β + tan α tan β

= 1 + 1 [From (i)]=2

Q.NO.3 Correct option: a 1,-1


𝟑
Q.NO.4 Correct option: d 𝟐
𝟏𝟐𝟏𝝅
Q.NO.5 Correct option: c 𝟓𝟒𝟎

ASSERTION-REASON BASED QUESTIONS

Q.NO.1 (d) If the radii of the two circles are r 1 and r2 and l is the length of the arc in either case , then
𝟑𝟎𝝅 𝟕𝟎𝝅
I = r1 ( circular measure of 30°) = r1( 𝟏𝟖𝟎 ) and l = r1 ( circular measure of 70°) = r1( 𝟏𝟖𝟎 )
𝒓𝟏𝝅 𝟕𝒓𝟐𝝅 𝒓 𝟕
= ⇒ 𝒓𝟏 = 𝟑
𝟔 𝟏𝟖 𝟐

𝒄𝒐𝒔(𝝅+𝒙).𝒄𝒐𝒔(−𝒙) −𝒄𝒐𝒔𝒙.𝒄𝒐𝒔 𝒙
Q.NO.2 (a) 𝝅 = 𝒔𝒊𝒏𝒙(−𝒔𝒊𝒏𝒙) =cot2x
𝒔𝒊𝒏(𝝅−𝒙)(𝒄𝒐𝒔 +𝒙)
𝟐

∴ cos(𝝅 + 𝜽) = -cos𝜽, cos(-𝜽) = cos 𝜽, sin(𝝅 − 𝜽) = sin𝜽 , sin(-𝜽) = sin 𝜽

2 MARKS QUESTIONS

Q.NO.1 Solution: Given, Length of the arc = l = 37.4 cm

Central angle = θ = 60° = 60π/180 radian = π/3 radians

We know that,r = l/θ (1 mark )


15

= (37.4) * (π / 3)= (37.4) / [22 / 7 * 3]= 35.7 cm (1 mark)

Q.NO.2 LHS cos 60° + cos 120° + cos 240° - sin 300°

cos 60° +cos (180° – 60°) +cos (270° – 30°) – sin (360°-60°) (1 mark)

cos 60° −cos 60° – cos 30° + sin 60°(1/2 mark)


√𝟑 √𝟑
- + =0 =RHS (1/2 mark)
𝟐 𝟐

𝟏+𝒔𝒊𝒏𝟐𝒙−𝐜𝐨𝐬 𝟐𝒙
Q.NO.3 LHS 𝟏+𝒔𝒊𝒏𝟐𝒙+𝒄𝒐𝒔𝟐𝒙

𝒔𝒊𝒏𝟐𝒙+𝟏−𝒄𝒐𝒔𝟐𝒙
(1/2 mark)
𝒔𝒊𝒏𝟐𝒙+𝟏+𝒄𝒐𝒔𝟐𝒙

𝟐𝒔𝒊𝒏𝒙𝒄𝒐𝒔𝒙+𝟐𝒔𝒊𝒏𝟐 𝒙
(1/2 mark)
𝟐𝒔𝒊𝒏𝒙𝒄𝒐𝒔𝒙+𝟐𝒄𝒐𝒔𝟐 𝒙

𝟐𝒔𝒊𝒏𝒙(𝒄𝒐𝒔𝒙+𝒔𝒊𝒏𝒙)
(1/2 mark)
𝟐𝒄𝒐𝒔𝒙(𝒔𝒊𝒏𝒙+𝒄𝒐𝒔𝒙)

𝒔𝒊𝒏𝒙
=tanx (1/2 mark)
𝒄𝒐𝒔𝒙

√𝟏−𝒔𝒊𝒏𝒙 √𝟏+𝒔𝒊𝒏𝒙
Q.NO.4 LHS +
√𝟏+𝒔𝒊𝒏𝒙 √𝟏−𝒔𝒊𝒏𝒙

𝟏−𝒔𝒊𝒏𝒙+𝟏+𝒔𝒊𝒏𝒙
(1 mark)
√𝟏−𝒔𝒊𝒏𝟐 𝒙

𝟐
(1/2 mark)
√𝒄𝒐𝒔𝟐 𝒙

𝟐
= secx(1/2 mark)
𝒄𝒐𝒔𝒙

Q.NO.5 LHS (cosx-cosy)2 + (sinx-siny)2

Cos2x+cos2y-2cosxcosy + sin2x+sin2y-2sinxsiny (1/2 mark)

Sin2x+cos2x+sin2y+cos2y -2(sinx siny +cosx cosy)(1/2 mark)

1+1 -2 cos(x-y) (1/2 mark)

2 – 2 cos(x-y)

2 {1 – cos (x-y)}
𝒙−𝒚
2 . 2 sin2( ) (1/2 mark)
𝟐
𝒙−𝒚
4 sin2( )
𝟐

3 MARKS QUESTIONS
𝜽 𝟗𝜽
Q.NO.1 LHS cos 2𝜽 cos𝟐 - cos 3𝜽 . 𝒄𝒐𝒔 𝟐

𝟏
Using formula cos A cos B = 𝟐 [ cos (A+B) + cos(A-B)] and
𝑨+𝑩 𝑨−𝑩
cos A – cos B = -2 sin sin
𝟐 𝟐
16

𝟏 𝜽 𝜽 𝟏 𝟗𝜽 𝟗𝜽
[ cos (2𝜽 +𝟐) + cos (2𝜽 - 𝟐 ) ] - 𝟐 [cos(3𝜽 + ) + cos(3𝜽 - 𝟐 )] (1 mark)
𝟐 𝟐

𝟏 𝟓𝜽 𝟑𝜽 𝟏𝟓𝜽 −𝟑𝜽
[ cos + cos - cos - cos( )]
𝟐 𝟐 𝟐 𝟐 𝟐

𝟏 𝟓𝜽 𝟑𝜽 𝟏𝟓𝜽 𝟑𝜽
[ cos 𝟐 + cos 𝟐 - cos - cos 𝟐 ] (1/2 mark)
𝟐 𝟐

𝟏 𝟓𝜽 𝟏𝟓𝜽
[ cos 𝟐 - cos ]
𝟐 𝟐
𝟓𝜽 𝟏𝟓𝜽 𝟓𝜽 𝟏𝟓𝜽
𝟏 + −
𝟐 𝟐 𝟐 𝟐
[ -2 sin sin ] (1 mark)
𝟐 𝟐 𝟐

𝟐𝟎𝜽 −𝟏𝟎𝜽
- sin sin( )
𝟒 𝟒

𝟓𝜽
Sin 5𝜽 sin 𝟐 = RHS proved(1/2 mark)

Q.NO.2 RHS cos2(A-B) +cos2B - 2 cos(A-B) cosA cosB

cos2B + cos2(A-B) – 2 cos (A-B) cosA cosB

cos2B + cos (A-B)[ cos(A-B) – 2cosA cosB] (1/2 mark)

cos2B + cos (A-B)[cosA cosB +sinA sinB – 2cosA cosB] (1mark)

cos2B + cos (A-B)[ sinA sinB - cosA cosB]

cos2B - cos (A-B) [cosA cosB - sinA sinB] (1/2 mark)

cos2B - cos (A-B)cos(A+B)

cos2B – (cos2A - sin2B) (1 mark)

cos2B +sin2B - cos2A

1 - cos2A= sin2A = LHS Proved(1/2 mark)


𝟑 −𝟏𝟐
Q.NO.3 sin A = 𝟓 cos B = 𝟏𝟑

where A and B both lie in second quadrant

𝟗 𝟒
cos A =±√𝟏 − 𝒔𝒊𝒏𝟐 𝑨 = ±√𝟏 − =± 𝟓(1/2 mark)
𝟐𝟓

𝟒
A lies in second quadrant so that cosA = - 𝟓(1/2 mark)

𝟏𝟒𝟒 𝟓
sin B=±√𝟏 − 𝒄𝒐𝒔𝟐 𝑩 =±√𝟏 − =± 𝟏𝟑(1/2 mark)
𝟏𝟔𝟗

𝟓
B lies in second quadrant so that sin B= 𝟏𝟑(1/2 mark)

Sin(A+B) = sinA cosB + cosA sinB(1/2 mark)


3 −12 𝟒 𝟓
=5 X + -𝟓 x (1/2 mark)
13 𝟏𝟑

−𝟑𝟔 𝟐𝟎 −𝟓𝟔
= - 𝟔𝟓 = 𝟔𝟓 (1/2 mark) ans
𝟔𝟓
17

Q.NO.4 LHS tan8x – tan 6x – tan 2x

tan 8x – (tan 6x+tan2x)


𝒕𝒂𝒏 𝟔𝒙+𝒕𝒂𝒏𝟐𝒙
by using formula tan8x = tan(6x+2x)=𝟏−𝒕𝒂𝒏𝟔𝒙 𝒕𝒂𝒏𝟐𝒙 (1 mark)

tan 8x (1-tan6x tan2x)=tan6x+tan2x

tan8x – [tan8x(1-tan6xtan2x)] (1/2 mark)

tan8x – [tan8x –tan8x tan6x tan2x] (1/2 mark)

tan8x – tan8x +tan8x tan6x tan2x(1/2 mark)

tan8x tan6x tan2x=RHS proved(1/2 mark)


𝐬𝐢𝐧 𝑨+𝐬𝐢𝐧 𝟑𝑨+𝒔𝒊𝒏𝟓𝑨+𝒔𝒊𝒏𝟕𝑨
Q.NO.5 LHS 𝐜𝐨𝐬 𝑨+𝒄𝒐𝒔𝟑𝑨+𝒄𝒐𝒔𝟓𝑨+𝒄𝒐𝒔𝟕𝑨
𝑨+𝟑𝑨 𝑨−𝟑𝑨 𝟓𝑨+𝟕𝑨 𝟓𝑨−𝟕𝑨
𝟐 𝒔𝒊𝒏( )𝒄𝒐𝒔( )+𝟐𝒔𝒊𝒏( )𝒄𝒐𝒔( )
𝟐 𝟐 𝟐 𝟐
= 𝑨+𝟑𝑨 𝑨−𝟑𝑨 𝟓𝑨+𝟕𝑨 𝟓𝑨−𝟕𝑨 (1 mark)
𝟐 𝒄𝒐𝒔( )𝒄𝒐𝒔( )+𝟐𝒄𝒐𝒔( )𝒄𝒐𝒔( )
𝟐 𝟐 𝟐 𝟐

𝟒𝑨 −𝟐𝑨 𝟏𝟐𝑨 −𝟐𝑨


𝟐 𝒔𝒊𝒏( )𝒄𝒐𝒔( )+𝟐𝒔𝒊𝒏( )𝒄𝒐𝒔( )
𝟐 𝟐 𝟐 𝟐
= 𝟒𝑨 −𝟐𝑨 𝟏𝟐𝑨 −𝟐𝑨
𝟐 𝒄𝒐𝒔( )𝒄𝒐𝒔( )+𝟐𝒄𝒐𝒔( )𝒄𝒐𝒔( )
𝟐 𝟐 𝟐 𝟐

𝒔𝒊𝒏𝟐𝑨 𝒄𝒐𝒔 (−𝑨)+𝒔𝒊𝒏𝟔𝑨𝒄𝒐𝒔(−𝑨)


= 𝒄𝒐𝒔𝟐𝑨𝒄𝒐𝒔(−𝑨)+𝒄𝒐𝒔𝟔𝑨𝒄𝒐𝒔(−𝑨) (1/2 mark)

𝒔𝒊𝒏𝟐𝑨 +𝒔𝒊𝒏𝟔𝑨
= 𝒄𝒐𝒔𝟐𝑨+𝒄𝒐𝒔𝟔𝑨
𝟐𝑨+𝟔𝑨 𝟐𝑨−𝟔𝑨
𝟐 𝒔𝒊𝒏( )𝒄𝒐𝒔( )
𝟐 𝟐
= 𝟐𝑨+𝟔𝑨 𝟐𝑨−𝟔𝑨 (1 mark)
𝟐 𝒄𝒐𝒔( )𝒄𝒐𝒔( )
𝟐 𝟐

𝒔𝒊𝒏𝟒𝑨𝒄𝒐𝒔(−𝟐𝑨)
=𝒄𝒐𝒔𝟒𝑨𝒄𝒐𝒔(−𝟐𝑨)

=tan4A = RHS proved(1/2 mark)

5 MARKS QUESTIONS
−𝟓
Q.NO.1 tanx = 𝟏𝟐 x lies in 2nd quadrant
𝝅
≤x≤𝝅
𝟐
𝝅 𝒙 𝝅
≤ 𝟐 ≤ 𝟐 (1/2 mark)
𝟒

1+tan2x = sec2x (1/2 mark)


−𝟓
1+(𝟏𝟐 )2 = sec2x
𝟐𝟓
1+𝟏𝟒𝟒 = sec2x
𝟏𝟔𝟗
= sec2x
𝟏𝟒𝟒

𝟏𝟑
± 𝟏𝟐 =secx(1/2 mark)
18

−𝟏𝟐
x lies in second quadrant cos x = 𝟏𝟑 (1/2 mark)

𝟏𝟐
𝒙 𝟏+𝒄𝒐𝒔𝒙 𝟏− 𝟏 𝟏
cos𝟐 = √ =√ 𝟐𝟏𝟑 = √𝟐𝟔 = (1 mark)
𝟐 √𝟐𝟔

𝟏𝟐
𝒙 𝟏−𝒄𝒐𝒔𝒙 𝟏+ 𝟐𝟓 𝟓
sin𝟐 =√ =√ 𝟐𝟏𝟑 = √𝟐𝟔 = (1 mark)
𝟐 √𝟐𝟔

𝒙
𝒙 𝒔𝒊𝒏
𝟐
tan𝟐 = 𝒙 =5 (1 mark)
𝒄𝒐𝒔
𝟐

𝝅 𝝅
Q.NO.2 LHS cos2x + cos2(x +𝟑 )+cos2(x - 𝟑 )
𝟏 𝝅 𝝅
[2cos2x +2 cos2x + cos2(x + 𝟑 )+cos2(x - 𝟑 )] (1 mark)
𝟐

𝟏 𝟐𝝅 𝟐𝝅
[( 1+cos2x)+1+cos(2x+ 𝟑 ) +1+ cos(2x - )] (1 mark)
𝟐 𝟑

𝟏 𝟐𝝅 𝟐𝝅
[3+cos2x +{ cos(2x+ 𝟑 ) + cos(2x - )] (1/2 mark)
𝟐 𝟑

𝟏 𝟐𝝅
[3+cos2x+2cos2x cos 𝟑 ] (1 mark)
𝟐

𝟏 −𝟏
[3+cos2x+2cos2x ( 𝟐 )] (1 mark)
𝟐

𝟏 𝟑
[3+cos2x - cos2x ] = (1/2 mark)
𝟐 𝟐

RHS

Q.NO.3 LHS cos 6x

=cos3(2x) (1/2 mark)

=4cos32x – 3cos2x (1 mark)

= 4[2cos2x-1]3 – 3 (2cos2x-1) (1/2 mark)

= 4 [ (2cos2x)3 -1 -3 . 2cos2x (2cos2x – 1)]-6cos2x+3 (1 mark)

=4[8cos6x – 1 -12 cos4x +6cos2x] – 6cos2x +3(1 mark)

=32cos6x – 4 – 48cos4x +24cos2x – 6cos2x +3 (1 mark)

=32cos6x – 48cos4x +18cos2x -1

CASE BASED QUESTIONS


𝟒 𝟑𝝅
Q.NO.1 Given , cosA= 𝟓, where < 𝑨 , B < 2𝝅
𝟐

𝟑
∴ sin A =−√𝟏 − 𝒄𝒐𝒔𝟐 𝑨 = − 𝟓 (1/2 mark )
𝟏𝟐
Cos B = 𝟏𝟑
𝟓
∴ sin B =−√𝟏 − 𝒄𝒐𝒔𝟐 𝑩 = − 𝟏𝟑 (1/2 mark )
19

𝟒 𝟏𝟐 𝟑 𝟓 𝟑𝟑
i) cos(A+B) = cos A cos B – sin A sin B = 𝟓 . 𝟏𝟑 -(− 𝟓 ).( − 𝟏𝟑) = 𝟔𝟓 ( 1 mark )
𝟑 𝟏𝟐 𝟒 𝟓 𝟏𝟔
ii) sin (A - B) = sin A cos B - cos A .sin B = − 𝟓 . 𝟏𝟑 - . (− 𝟏𝟑 ) = 𝟔𝟓 ( 1 mark )
𝟓

𝟑 𝟏𝟐 𝟒 𝟓 𝟓𝟔
iii) sin (A + B) = sin A cos B + cos A .sin B = − 𝟓 . 𝟏𝟑 + 𝟓 . (− 𝟏𝟑 ) = − 𝟔𝟓 ( 1 mark )
−𝟓𝟔
𝒔𝒊𝒏 (𝑨+𝑩) 𝟔𝟓 𝟓𝟔
tan ( A + B ) = 𝒄𝒐𝒔 ( 𝑨+𝑩 ) = 𝟑𝟑 = − 𝟑𝟑 ( 1 mark )
𝟔𝟓

𝝅
Q.NO.2 A + B = 𝟒 ,
𝝅
tan(A + B) = 𝒕𝒂𝒏 ,
𝟒

𝒕𝒂𝒏 𝑨+𝒕𝒂𝒏 𝑩
𝒊) 𝟏−𝒕𝒂𝒏 𝑨 𝒕𝒂𝒏 𝑩 = 1

tan A + tan B = 1 – tan A tan B

tan A + tan B + tan A tan B =1 ( eq no 1)

1 + tan A + tan B + tan A tan B =1 + 1

(1 + tan A ) + tan B (1 + tan A) = 2

(1 + tan A ) (1 + tan B ) = 2

ii) on dividing both sides of eq 1 bby tan A tan B ,


𝒕𝒂𝒏 𝑨+𝒕𝒂𝒏 𝑩+𝒕𝒂𝒏 𝑨 𝒕𝒂𝒏 𝑩 𝟏
= 𝒕𝒂𝒏 𝑨 𝒕𝒂𝒏 𝑩
𝒕𝒂𝒏 𝑨 𝒕𝒂𝒏 𝑩

cot B + cot A + 1= cot A cot B

cot A cot B - cot B - cot A = 1

cot A cot B - cot B - cot A + 1= 2

cot A ( cot B – 1 ) – ( cot B – 1 ) = 2

( cot B – 1 ) (cot A – 1 ) = 2
𝝅
iii) A + B = 𝟒 ,
𝝅
sin (A + B ) = sin 𝟒
𝟏
sin (A + B ) =
√𝟐
𝝅
cos (A + B ) = cos 𝟒
𝟏
cos (A + B ) =
√𝟐
𝝅
tan(A + B) = 𝒕𝒂𝒏 𝟒
tan(A + B) = 1
𝟏 𝟏
sin(A+B)-cos(A+B)+tan(A+B) = - +1 = 1
√𝟐 √𝟐
20

(Chapter -4: COMPLEX NUMBERS)


MCQs
1. If a + ib = c + id, then

(a) a2 + c2 = 0 (b) b2 + c2 = 0 (c) b2 + d2 = 0 (d) a2 + b2 = c2 + d2

2. If 1 – i, is a root of the equation x2 + ax + b = 0, where a, b ∈ R, then the value of a – b is

(a) -4 (b) 0 (c) 2 (d) 1

3. If [(1 + i)/(1 – i)]x = 1, then


(a) x = 2n + 1, where n ∈ N (b) x = 4n, where n ∈ N
(c) x = 2n, where n ∈ N (d) x = 4n + 1, where n ∈ N

4. The simplified value of (1 – i)3/(1 – i3) is

(a) 1 (b) -2 (c) –I (d) 2i

5. sin x + i cos 2x and cos x – i sin 2x are conjugate to each other for:

(a) x = nπ (b) x = [n + (1/2)] (π/2) (c) x = 0 (d) No value of x

=========================================================

Answers with explanation


Ans . Q.1. (d) a2 + b2 = c2 + d2

a + ib = c + id

⇒ |a + ib| = |c + id|

⇒ √(a2 + b2) = √(c2 + d2)

Squaring on both sides, we get;

a2 + b2 = c2 + d2

Ans. Q.2. Correct option: (a) -4

Given that 1 – i is the root of x2 + ax + b = 0.

Thus, 1 + i is also the root of the given equation since non-real complex roots occur in conjugate pairs.

Sum of roots = −a/1 = (1 – i) + (1 + i)

⇒a=–2

Product of roots, b/1 = (1 – i)(1 + i)

b = 1 – i2

b = 1 + 1 {since i2 = -1}
21

⇒b=2

Now, a – b = -2 – 2 = -4

Ans. Q.3. Correct option: (b) x = 4n, where n ∈ N

Solution: Given,
[(1 + i)/(1 – i)]x = 1
By rationalizing the denominator,
[(1 + i)(1 + i)/ (1 – i)(1 + i)]x = 1
[(1 + i)2/ (1 – i + i – i2)]x = 1
[(1 + i2 + 2i)/(1 + 1)]x = 1
[(1 – 1 + 2i)/ 2]x = 1
ix = 1
Thus, ix = i4n, where n is any positive integer.

Ans Q.4. Correct option: (b) -2

Solution:

(1 – i)3/(1 – i3)

= (1 – i)3/(13 – i3)

= (1 – i)3/ [(1 – i)(1 + i + i2)]

= (1 – i)2/(1 + i – 1)

= (1 – i)2/i

= (1 + i2 – 2i)/i

= (1 – 1 – 2i)/i

= -2i/i

= -2

Ans Q.5. Correct option: (d) No value of x

Solution:

Consider sin x + i cos 2x and cos x – i sin 2x are conjugate to each other.

So, sin x – i cos 2x = cos x – i sin 2x

On comparing real and imaginary parts of both sides, we get

⇒ sin x = cos x and cos 2x = sin 2x

⇒ sin x/cos x = 1 and (cos 2x/sin 2x) = 1

⇒ tan x = 1 and tan 2x = 1

Now, consider tan 2x = 1


22

Using the formula tan 2A = 2 tan A/(1 – tan2A),

(2 tan x)/(1 – tan2x) = 1

However, this is not possible for tan x = 1.

Therefore, for no value of x, sinx + i cos 2x and cos x – i sin 2x are conjugate to each other.

=====================================================================

ASSERTION REASONING QUESTIONS

Instructions : Each question consists of two statements, namely, Assertion (A) and Reason (R).For selecting the
correct answer, use the following code:
(a) Both Assertion (A) and Reason (R) are the true and Reason (R) is a correct explanation of Assertion (A).
(b) Both Assertion (A) and Reason (R) are the true but Reason (R) is not a correct explanation of Assertion (A).
(c) Assertion (A) is true and Reason (R) is false.
(d) Assertion (A) is false and Reason (R) is true.

Q.1. Assertion(A) : If z1= 2+3i and z2 = 3-2i, then z1 – z2 = -1+5i


Reason (R) : If z1 = a + ib and z2 = c + id, then z1 – z2 = (a-c) + i(b-d)

Q.2. Assertion(A) : If (1 + i )(x + iy) = 2-5i, then x = -3/2 and y = -7/2

Reason (R) : if a + ib = c + id, then a=c and b = d

Q.3. Assertion(A) : The multiplicative inverse of 2 – 3i is 2 +3i

Reason (R) : If z = 3+4i, then 𝑧̅ = 3-4i

Answers
Q.1. (a) Both Assertion (A) and Reason (R) are the true and Reason (R) is a correct explanation of Assertion (A).

Q.2. (a) Both Assertion (A) and Reason (R) are the true and Reason (R) is a correct explanation of Assertion (A).

Q.3. (d) Assertion (A) is false and Reason (R) is true.

==============================================================================
VSA Type Questions (2 marks each)
Q.1. Write the complex number i9 + i19 in the form of a + ib.

Q.2. Express the given expression (1 + i) (1 + 2i) in the form a + ib and find the values of a
and b.

Q.3. Determine the multiplicative inverse of 4 – 3i.

Q.4. Find the modulus of the complex number : (1 + 2i) / (1 – 3i).

1+𝑖 𝑚
Q.5. If ( ) = 1, find the least positive integral value of m.
1−𝐼

Answers
Sol Q(1) : Given number: i9 + i19.
23

The expression i9 + i19 can be represented as follows:


i9 + i19 = (i2)4. i + (i2)9. i …(1)
We know that, i2 = 1.
On substituting i2 = -1, we get
i9 + i19 = (-1)4.i + (-1)9.i
i9 + i19 = 1.i + (-1).i
i9 + i19 = i – i
i9 + i19 = 0.
Therefore, i9 + i19 in the form of a + ib is 0 + i0.
Sol Q(2) : Given expression: (1 + i) (1 + 2i)
Hence, (1 + i) (1 + 2i) = 1(1) + 1(2i) + i + 2i(i)
(1 + i) (1 + 2i) = 1 + 2i + i + 2i2
(1 + i) (1 + 2i) = 1 + 2i + i + 2(-1) [As, i2 = -1]
(1 + i) (1 + 2i) = 1 + 2i + i – 2
(1 + i) (1 + 2i) = -1 + 3i
Hence, the expression (1 + i) (1 + 2i) in the form of a + bi is -1 + 3i.
Thus, the value of a = -1 and b = 3.
Sol Q(3) : Let z = 4 – 3i.
As we know, the multiplicative inverse of z is 1/z.
Hence, 1/z = 1/ (4+3i)
Therefore, the multiplicative inverse of 4 – 3i is:
1 4+ 3i
= 4 – 3i × 4+ 3i
4+ 3i
= 16+9
4+ 3i
= 25

Therefore, the multiplicative inverse of 4 – 3i is (4 + 3i)/25.


1+2𝑖 |1+2𝑖| √12 +22 √5 1
Sol Q(4) : |1−3𝑖| = = = =
|1−3𝑖| √12 +(−3)2 √10 √2
24

𝟏+𝒊 𝒎
Sol Q(5) ( ) =1
𝟏−𝑰

𝟏+𝒊 𝟏+𝒊 𝒎
 (𝟏−𝑰 × 𝟏+𝒊) = 𝟏

𝟏+𝟐𝒊−𝟏 𝒎
 ( ) =𝟏
𝟏+𝟏

 (𝒊𝒎 = 𝟏)

Therefore, the least positive integral value of m is 4.

===========================================================================
SA Type Questions (3 marks each)
Q.1. If z1 and z2 are the two complex numbers, then show that :

Re (z1z2) = Re z1 Re z2 – Im z1 Im z2.

Q.2. Compute the value of (1-i)n [1- (1/i)]n for a positive integer “n”.

Q.3. Express in the form of a + bi : (1/3 + 3i)3

Q.4. Express in the form of a + ib: (3i – 7) + (7 – 4i) – (6 + 3i) + i23

Q.5. The complex numbers sin x + i cos 2x is the conjugate to itself. Find the value of x.

Answers
Sol Q(1) : Let z1 = a1 + ib1 and z2 = a2+ ib2
Now, take the product of these two complex numbers.
z1z2 = (a1 + ib1)(a2+ ib2)
z1z2 = a1(a2+ ib2) + ib1(a2+ ib2)
z1z2 = a1a2 + ia1b2 + ia2b1 + i2b1b2
z1z2 = a1a2 + ia1b2 + ia2b1 – b1b2 [since i2 = -1]
Now, the above equation can be rearranged as follows:
z1z2 = a1a2 – b1b2 + ia1b2 + ia2b1
z1z2 = (a1a2 – b1b2) + i(a1b2 + a2b1)
Here, the real part is:
Re (z1z2) = a1a2 – b1b2
Thus,
Re (z1z2) = Re z1 Re z2 – Im z1 Im z2.
Hence, Re (z1z2) = Re z1 Re z2 – Im z1 Im z2 is proved.
25

Sol Q(2) : Given: (1 – i)n [1- (1/i)]n


We know that i4 = 1,
Hence, (1 – i)n [1- (1/i)]n = (1 – i)n [1- (i4/i)]n
(1 – i)n [1- (1/i)]n = (1 – i)n [1 – i3]n
Further, the above equation is written as follows:
(1 – i)n [1- (1/i)]n = (1 – i)n (1 + i)n [Since i3 = -i]
(1 – i)n [1- (1/i)]n = [(1 – i) (1 + i)]n
The expression (1 – i) (1 + i) is of the form (a – b)(a + b), which is equal to a2 – b2.
Here, a = 1, b = 1
Thus, (1 – i)n [1- (1/i)]n = (1 – i2)n
(1 – i)n [1- (1/i)]n = ( 1 – (-1))n [As, i2 = -1]
(1 – i)n [1- (1/i)]n = (1 + 1)n
(1 – i)n [1- (1/i)]n = 2n.
Therefore, the value of (1 – i)n [1- (1/i)]n is 2n.
Sol Q(3) : (1/3 + 3i)3 = (1/3)3 + (3i)3 + 3(1/3) (3i) (1/3 +3i)

= (1/27) + 27i3 + 3i(1/3 +3i)

= 1/27 + 27(-i) + i + 9i2 [i3 = -i]

= 1/27 – 27i + i – 9 [ i2 = -1]

= (1/27 – 9) + i(-27 + 1)

= -242/27 – 26i

Sol Q(4) : (3i – 7) + (7 – 4i) – (6 + 3i) + i23

= 3i – 7 + 7 – 4i – 6 – 3i + i23

= -4i – 6 + i22+1

= -4i – 6 + (i2)11 . i

= -4i – 6 + (-1)11 . i

= -4i – 6 – i

Thus, (3i – 7) + (7 – 4i) – (6 + 3i) + i23 = -6 – 5i

Sol Q(5) : Complex number = sin x + i cos 2x (given)

Conjugate number = sin x – i cos 2x

Now, sin x + i cos 2x = sin x – i cos 2x


26

⇒ sin x = cos x and sin 2x = cos 2x [comparing real and imaginary part]

⇒ tan x = 1

and tan 2x = 1

Now, both of them are not possible for the same value of x.

Therefore, x has no value.

===========================================================================LA
Type Questions (5 marks each)
β−α
Q.1. If αandβ are different complex numbers with |β| = 1, thenfind |1−α̅β|
𝑢 𝑣
Q.2. If (x+iy)3 =u+iv, where u,v,x,y R, then show that 𝑥 + 𝑦 = 4(𝑥 2 − 𝑦 2 )

(1+𝑖)𝑥−2𝑖 (2−3𝑖)𝑦+𝑖
Q.3. Find x and y , if + = 𝑖, 𝑤ℎ𝑒𝑟𝑒 𝑖 = √−1 𝑎𝑛𝑑 𝑥, 𝑦 ∈ 𝑅
3+𝑖 3−𝑖

Answers

Sol.Q.1. we know that |𝒛|𝟐 = 𝒛. 𝒛̅

𝜷−𝜶 𝟐 𝜷−𝜶 ̅̅̅̅̅


𝜷−𝜶
 |
𝟏−α̅ β
| =(
𝟏−α̅ β
)( )
𝟏−α̅ β

𝜷−𝜶 ̅ −𝜶
𝜷 ̅
= (𝟏−α̅ β) (𝟏−𝜶𝜷̅) using properties of conjugate
̅ +|𝛼|2
̅ 𝛽−𝛼𝛽
1−𝛼
= ̅ +|𝛼|2
̅ 𝛽−𝛼𝛽
simplifying and using |𝒛|𝟐 = 𝒛. 𝒛̅ as well as |𝜷| = 𝟏
1−𝛼

=1
𝜷−𝜶
 |𝟏−α̅ β| = 𝟏 proved .

Sol.Q.2. Given, (x+iy)3=u+iv


 x3 + (iy)3 + 3.x.iy(x+iy) = u + iv
 (x3−3xy2) + i(3x2y−y3) = u + iv

on equating real and imaginary parts , we get


u=x3−3xy2,v=3x2y−y3
𝒖 𝒗
 𝒙 = x2 -3y2 and 𝒚 = 3x2 –y2
𝑢 𝑣
 𝑥 + 𝑦 = 4(𝑥 2 − 𝑦 2 )
(1+𝑖)𝑥−2𝑖 (2−3𝑖)𝑦+𝑖
Sol.Q.3. Given equation is + =𝑖
3+𝑖 3−𝑖

simplifying and comparing the real and imaginary parts from both sides, we get
4x + 9y - 3 = 0 ----- (1)
27

2x−7y−13=0 ----(2)
Solving (1) and (2) , we get
x = 3 and y = -1
===============================================================================
CASE Based Questions
Q.1. A complex number z is said to be purely real iff 𝑧̅ = 𝑧 and purely imaginary iff 𝑧̅ = −𝑧, where 𝑧̅ is the
conjugate of z. If z = x + iy , then 𝑧̅ = 𝑥 − 𝑖𝑦.
Based on the above information, answer the following questions :
(i) Which of the followings is not true
̅̅̅̅̅
𝑧 ̅̅̅
𝑧1
(a) ̅̅̅̅̅̅̅̅̅
𝑧1 + 𝑧2 = 𝑧̅1 + 𝑧̅2 (b) ̅̅̅̅̅̅
𝑧1 𝑧2 = 𝑧̅1 𝑧̅2 (c) ̅̅̅̅̅̅̅̅
𝑧1 −𝑧2 = 𝑧̅2 − 𝑧̅1 (d) (𝑧1 ) = ̅̅̅
1 mark
2 𝑧2

(ii) If 𝑧1 = 1 + 𝑖 𝑎𝑛𝑑 𝑧2 = 2 − 𝑖, 𝑡ℎ𝑒𝑛 ̅̅̅̅̅̅


𝑧1 𝑧2 = ?

(a) 3-i (b) i-3 (c) 3 + i (d) none of these 1 mark


3+2𝑖𝑆𝑖𝑛𝜃
(iii) If z = 1−2𝑖𝑆𝑖𝑛𝜃 (0 < 𝜃 ≤ 𝜋/2) is purely imaginary , then 𝜃 is equal to

(a) 𝜋/4 (b) 𝜋/6 (c) 𝜋/3 (d) 𝜋/12 2 marks

Q.2. Two complex numbers z1 = a + ib and z2 = c+ id are said to be equal iff a = c and b = d.

On the basis of above information, answer the following questions :

(i) If (x+y) + i(x-y) = 4 + 6i, then xy = ?

(a) 5 (b) -5 (c) 4 (d) -4


(𝑖+𝑖)2
(ii) If = 𝑥 + 𝑖𝑦, then the value of x + y is
2−𝑖

(a) 1/5 (b) 3/5 (c) 4/5 (d) 2/5

Answers

Q.1. (i) a (ii) c (iii) c

Q.2. (i) b (ii) d


28

(Chapter -5: LINEAR INEQUALITY)


LINEAR INEQUALITY
Q1) If | x – 1| > 5, then

(a) x ∈ (–4, 6)

(b) x ∈ [–4, 6]

(c) x ∈ (–∞, –4) ∪ (6, ∞)

(d) x ∈ [–∞, –4) ∪ [6, ∞)


Q2) If x < 5, then
(a) –x < –5
(b) –x ≤ –5
(c) –x > –5
(d) –x ≥ –5
Q3) If –3x + 17 < –13, then

(a) x ∈ (10, )

(b) x ∈ [10, )

(c) x ∈ (– , 10]

(d) x ∈ [–10, 10)


Q4)The inequality representing following graph is

a) | x | < 5 (b) | x | ≤ 5 (c) | x | > 5 (d) | x | ≥ 5


Q5) If x, y and b are real numbers and x < y, b < 0, then
𝑥 𝑦
a) <𝑏
𝑏
𝑥 𝑦
b) >𝑏
𝑏
𝑥 𝑦
c) ≥𝑏
𝑏
𝑥 𝑦
d) ≤𝑏
𝑏

2 MARKS
Q6) Draw the graphical solution of the following inequation : x > 3
Q7) Solve for x : 3x – 4 ≥ – 2x + 6 when (i) x is a natural number (ii) x is a real number.
29

Q8) Solve : 3x + 5 < x – 7, when (i) x is an integer (ii) x is a real number.


Q9) Draw the graphical solution of the following system of inequation :
4−3𝑋 2𝑋−5
<
5 4

Q10) Solve – 12x > 30, when (i) x is a natural number (ii) x is an integer.

3 MARKS
2−3𝑋
Q11) Solve :-5≤ ≤9
4

Q12) Solve : 5 (2x – 7) –3 (2x + 3) ≤ 0 and 2x + 19 ≤ 6x + 47 and represent the solution on number line.
4−7𝑋
Q13) Draw the graphical solution of the following system of inequation : -3≤ ≤18
2

𝑀𝐸𝑁𝑇𝐴𝐿 𝐴𝐺𝐸
Q14) I.Q. of a person is given by formula I.Q. = 𝐶𝐻𝑅𝑂𝑁𝑂𝐿𝑂𝐺𝐼𝐶𝐴𝐿 𝐴𝐺𝐸 × 100, where M.A. stands for mental age
and C.A., stands for chronological age. If 75 ≤ I.Q. ≤ 135 for a group of 9 year children. Find the range of
their mental age.

Q15) Given set A = {x ∈ Z : 0 ≤ x ≤ 10}. Solve the inequation –2x + 6 ≤ 5x – 4 in set A.

CASE BASE 4 MARKS


Q16

The marks of four students out of 100 in 4 test are given below and the grading scheme is also given.
Read the given information carefully and answer the following.

Name Test-1 Test-2 Test-3 Test-4

Prachi 85 93 94 89

Reshma 75 86 76 75

Ankit 92 83 44 60

Sunil 59 81 62 73

Grading system
Average Marks (x) Grade
x ≥ 91 A1
90 ≥ x ≥ 81 A2
80 ≥ x ≥ 71 B1
70 ≥ x ≥ 61 B2
60 ≥ x ≥ 51 C
(i) To get grade A1 the minimum marks Prachi should score in test-5 is
(a) 94 (b) 95
30

(c) 90 (d) ≥ 94
(ii) The marks Reshma should score in Test-5 to get grade A2 is
(a) ≥ 95 (b) 93
(c) ≥ 93 (d) 90
(iii) If Ankit scored 91 marks in his test-5, then he will get grade
(a) A2 (b) B1
(c) B2 (d) C
(iv) To get average marks more than Ankit, the minimum marks Sunil should score is
(a) 96 (b) 95
(c) 98 (d) 97

Q1717 A manufacturer has 640 liters of 8% boric acid solution and 1300 liters of 2% boric acid solution. To
dilute the 8% boric acid solution manufacturer added some amount of 2% boric acid solution into it.
(i) The maximum amount of 2% boric acid solution to be added so that resulting mixture is to be
more than 4% boric acid is
(a) 1280 l (b) 1278 l
(c) 1300 l (d) 1200 l
(ii) The maximum amount of 2% boric acid solution left after adding to 8% boric acid solution so
that resulting mixture to be less than or equal to 6% boric acid is
(a) 20 l (b) 800 l
(c) 980 l (d) 1000 l
(iii) If manufacturer used water in place of 2% boric acid solution for dilution of 8% boric acid
solution, then amount of water to be added so that resulting mixture is more than 2% boric acid is
(a) ≤ 1920 l (b) 3840 l
(c) 1920 l (d) <1920 l
(iv) The minimum amount of water to be added to 8% boric acid solution so that resulting mixture
is less than 4% boric acid solution is
(a) 641 l (b) 640 l
(c) 648 l (d) 650 l
31

18 A company produces certain items. The manager in the company used to make data record on daily basis
about the cost and revenue of these items separately. The cost function is given by C(x) = 40x + 14000 and
the revenue function is given by R(x) = 80x + 6000, where x is the number of items produced and sold
considering that all items produced are sold.
(i) Number of items produced to get some profit is
(a) > 200 (b) 200
(c) ≥ 200 (d) > 102
(ii) If the cost function becomes C(x) = 40x + 12000 and revenue function remains same then number of
items produced to get some profit is
(a) ≥ 150 (b) > 150
(c) > 200 (d) ≤ 150
(iii) To get at least ₹ 26,000 revenue per day the minimum number of items produced and sold per day is
(a) 255 (b) 257
(c) 250 (d) 300
(iv) The profit company will get after producing minimum number of items to get at least 26,000
revenue per day
(a) 2,400 (b) 20,000
(c) 24,000 (d) 2,000
(v) If the company has only ₹ 22,000 to invest in producing items per day. The maximum number of
items that can be produced is
(a) 250 (b) ≥ 200
(c) 200 (d) ≤ 200

5 MARKS
Q19)A manufacturer has 600 litres of a 12% solution of acid. How many litres of a 30% acid solution must
be added to it so that acid content in the resulting mixture will be more than 15% but less than 18% ?
Q20)A company manufactures cassettes. Its cost and revenue functions are C(x) = 26,000 + 30x and R(x) =
43x, respectively, where x is the number of cassettes produced and sold in a week. How many cassettes
must be sold by the company to realise some profit ?
Q21)The water acidity in a pool is considered normal when the average pH reading of three daily
measurements is between 8.2 and 8.5. If the first two pH readings are 8.48 and 8.35, find the range of pH
value for the third reading that will result in the acidity level being normal.

Assertion and reasoning


𝑎 𝑏
Q1) Assertion : If a<b,c<0 then 𝑐 < 𝑐 ,

Reasoning:if both sids are divided by same negative quantity, then inequality is reversed
32

Q2) Assertion: the line divide the cartisian plane in two parts

Reasoning:if a point P(a,b) lie on line dx+ey =f then da+eb=f .

(Chapter -6: PERMUTATION AND COMBINATION)


Q -1 In how many ways 5 girls and 3 boys be seated in a row, so that no two boys are together?

1. 1440
2. 14400
3. 3600
4. None of the above

Q-2 If 2nC3 : nC2 = 12 : 1, then the value of n is ?

1. 6
2. 5
3. 4
4. None of the above

Q-3 In how many ways a committee, consisting of 5 men and 6 women can be formed from 8 men and
10 women?

1. 266
2. 5040
3. 11760
4. None of the above

Q-4 In how many different ways can the letters of the word 'GEOGRAPHY' be arranged such that the
vowels must always come together?

1. 2520
2. 2530
3. 15130
4. 15120

Q-5 In how many ways can the word CHRISTMAS be arranged so that the letters C and M are never
adjacent?

1. 8! × (7/2)
2. 9! × (7/2)
3. 8! × (9/2)
4. None of the above
Q-1 2

Q-2 2

Q-3 3
33

Q-4 4

Q-5 1

Assertion and Reason Questions


Directions: Each of these questions contains two statements, Assertion and Reason. Each of these
questions also has four alternative choices, only one of which is the correct answer. You have to select
one of the codes (a), (b), (C) and (d) given below.
(a) Assertion is correct, reason is correct; reason is a correct explanation for assertion.
(b) Assertion is correct, reason is correct; reason is not a correct explanation for assertion
(c) Assertion is correct, reason is incorrect
(d) Assertion is incorrect, reason is correct.
1. Assertion: If the letters W, I, F, E are arranged in a row in all possible ways and the words (with or
without meaning) so formed are written as in a dictionary, then the word WIFE ocurs in the 24"
position.
Reason: The number of ways of arranging four distinct objects taken all at a time is C(4,4).
2. Assertion: A number of four different digits is formed with the help of the digits 1,2,3,4,5,6,7 in all
possible ways. Then, number of ways which are exactly divisible by 4 is 200.
Reason: A number divisible by 4, if unit place digit is divisible by 4
3. Assertion: Product of five consecutive natural numbers is divisible

2 MARKS QUESTION

Question 1:

Find the 3-digit numbers that can be formed from the given digits: 1, 2, 3, 4 and 5 assuming that

a) digits can be repeated.

b) digits are not allowed to be repeated.

Solution:

a) By the multiplication principle, the number of ways in which three-digit numbers can be formed
from the given digits is 5 × 5 × 5 = 125

b) By the multiplication principle, the number of ways in which three-digit numbers can be formed
without repeating the given digits is 5 × 4 × 3 = 60

Question 2:

A coin is tossed 6 times, and the outcomes are noted. How many possible outcomes can be there?

Solution:

When we toss a coin once, the number of outcomes we get is 2 (Either Head or tail)

So, in each throw, the no. of ways to get a different face will be 2.

Therefore, by the multiplication principle, the required no. of possible outcomes is

2 x 2 x 2 x 2 × 2 × 2 = 64
34

Question 3:

Evaluate the following

(i) 6 ! (ii) 5 ! – 2 !

Solution:

(i) 6! = 1 × 2 × 3 × 4 × 5 × 6 = 720

(ii) 5! = 1 × 2 × 3 × 4 x 5 = 120

As 2! = 1 × 2 = 2

Therefore, 5 ! – 2 ! = 120-2 = 118.

Question 4:

From a team of 6 students, in how many ways can we choose a captain and vice-captain assuming
one person can not hold more than one position?

Solution:

From a team of 6 students, two students are to be chosen in such a way that one student will hold
only one position.

Here, the no. of ways of choosing a captain and vice-captain is the permutation of 6 different things
taken 2 at a time.

So, 6P2 = 6! / ( 6 -2 )! = 6! / 4! = 30

Question 5:

How many words, with or without meaning, can be formed using all the letters of the word
EQUATION, using each letter exactly once?

Solution:

Number of letters in word EQUATION` = 8

n=8

If all letters of the word used at a time

r=8

Different numbers formed = nPr

= 8P 8

= 8!/(8 8)!

= 8!/0!

= 8!/1

= 8!
35

=8*7*6*5*4*3*2*1

= 40320

3 MARKS QUESTION

1. How many numbers are there between 99 and 1000, having at least one of their digits 7?

Solution:

Numbers between 99 and 1000 are all three-digit numbers.

Total number of 3 digit numbers having at least one of their digits as 7 = (Total numbers of three-digit
numbers) – (Total number of 3 digit numbers in which 7 does not appear at all)

= (9 × 10 × 10) – (8 × 9 × 9)

= 900 – 648

= 252

2. How many 5-digit telephone numbers can be constructed using the digits 0 to 9, if each number starts
with 67 and no digit appears more than once?

Solution:

Let ABCDE be a five-digit number.

Given that the first two digits of each number are 6 and 7.

Therefore, the number is 67CDE.

As repetition is not allowed and 6 and 7 are already taken, the digits available for place C are 0, 1, 2, 3, 4, 5, 8,
9, i.e. eight possible digits.

Suppose one of them is taken at C, now the digits possible at place D is 7.

Similarly, at E, the possible digit is 6.

Therefore, the total five-digit numbers with given conditions = 8 × 7 × 6 = 336.

3. Find the number of permutations of the letters of the word ALLAHABAD.

Solution:

Given word – ALLAHABAD

Here, there are 9 objects (letters) of which there are 4As, 2 Ls and rest are all different.

Therefore, the required number of arrangements = 9!/(4! 2!)

= (1 × 2 × 3 × 4 × 5 × 6 × 7 × 8 × 9)/ (1 × 2 × 3 × 4 × 1 × 2)
36

= (5 × 6 × 7 × 8 × 9)/2

= 7560

4. In how many of the distinct permutations of the letters in MISSISSIPPI do the four Is not come
together?

Solution:

Given word – MISSISSIPPI

M–1

I–4

S–4

P–2

Number of permutations = 11!/(4! 4! 2!) = (11 × 10 × 9 × 8 × 7 × 6 × 5 × 4!)/ (4! × 24 × 2)

= 34650

We take that 4 I’s come together, and they are treated as 1 letter,

∴ Total number of letters=11 – 4 + 1 = 8

⇒ Number of permutations = 8!/(4! 2!)

= (8 × 7 × 6 × 5 × 4!)/ (4! × 2)

= 840

Therefore, the total number of permutations where four Is don’t come together = 34650 – 840 = 33810
5. In a small village, there are 87 families, of which 52 families have at most 2 children. In a rural
development programme, 20 families are to be chosen for assistance, of which at least 18 families must
have at most 2 children. In how many ways can the choice be made?

Solution:

Given,

Total number of families = 87

Number of families with at most 2 children = 52

Remaining families = 87 – 52 = 35

Also, for the rural development programme, 20 families are to be chosen for assistance, of which at least 18
families must have at most 2 children.

Thus, the following are the number of possible choices:

52C18 × 35C2 (18 families having at most 2 children and 2 selected from other types of families)
37

52C19 × 35C1 (19 families having at most 2 children and 1 selected from other types of families)

52C20 (All selected 20 families having at most 2 children)

Hence, the total number of possible choices = 52C18 × 35C2 + 52C19 × 35C1 + 52C20

5 MARKS QUESTION
1. A committee of 3 persons is to be constituted from a group of 2 men and 3 women. In how many ways
can this be done? How many of these committees would consist of 1 man and 2 women?

Solution:

Given,

Men = 2

Women = 3

A committee of 3 persons to be constituted.

Here, the order does not matter.

Therefore, we need to count combinations.

There will be as many committees as combinations of 5 different persons taken 3 at a time.

Hence, the required number of ways = 5C3

= 5!/(3! 2!)

= (5 × 4 × 3!)/(3! × 2)

= 10

Committees with 1 man and 2 women:

1 man can be selected from 2 men in 2C1 ways.

2 women can be selected from 3 women in 3C2 ways.

Therefore, the required number of committees = 2C1 × 3C2

= 2 × 3C1

=2×3

=6

2. Determine the number of 5 card combinations out of a deck of 52 cards, if there is exactly one ace in
each combination.

Solution:
38

Given a deck of 52 cards

There are 4 Ace cards in a deck of 52 cards.

According to the given, we need to select 1 Ace card out of the 4 Ace cards

∴ The number of ways to select 1 Ace from 4 Ace cards is 4C1

⇒ More 4 cards are to be selected now from 48 cards (52 cards – 4 Ace cards)

∴ The number of ways to select 4 cards from 48 cards is 48C4

Number of 5 card combinations out of a deck of 52 cards if there is exactly one ace in each combination =
4C1 × 48C4

= 4 × [48!/(44! 4!)]

= 4 × [(48 × 47 × 46 × 45 × 44!)/ (44! × 24)]

= 4 × 2 × 47 × 46 × 45

= 778320

3. A group consists of 4 girls and 7 boys. In how many ways can a team of 5 members be selected if the
team has

(i) no girls

(ii) at least one boy and one girl

(iii) at least three girls

Solution:

Given,

Number of girls = 7

Number of boys = 7

(i) No girls

Total number of ways the team can have no girls = 4C0 × 7C5

= 1 × 21

= 21

(ii) at least one boy and one girl

1 boy and 4 girls = 7C1 × 4C4 = 7 × 1 = 7

2 boys and 3 girls = 7C2 × 4C3 = 21 × 4 = 84


39

3 boys and 2 girls = 7C3 × 4C2 = 35 × 6 = 210

4 boys and 1 girl = 7C4 × 4C1 = 35 × 4 = 140

Total number of ways the team can have at least one boy and one girl = 7 + 84 + 210 + 140

= 441

(iii) At least three girls

Total number of ways the team can have at least three girls = 4C3 × 7C2 + 4C4 × 7C1

= 4 × 21 + 7

= 84 + 7

= 91

9. How many numbers greater than 1000000 can be formed using the digits 1, 2, 0, 2, 4, 2, 4?

Solution:

Given numbers – 1000000

Number of digits = 7

The numbers have to be greater than 1000000, so they can begin either with 1, 2 or 4.

When 1 is fixed at the extreme left position, the remaining digits to be rearranged will be 0, 2, 2, 2, 4, 4, in
which there are 3, 2s and 2, 4s.

Thus, the number of numbers beginning with 1 = 6!/(3! 2!) = (6 × 5 × 4 × 3!)/(3! × 2)

= 60

The total numbers begin with 2 = 6!/(2! 2!) = 720/4 = 180

Similarly, the total numbers beginning with 4 = 6!/3! = 720/6 = 120

Therefore, the required number of numbers = 60 + 180 + 120 = 360.

10. 18 mice were placed in two experimental groups and one control group, with all groups equally large. In
how many ways can the mice be placed into three groups?

Solution:

Number of mice = 18

Number of groups = 3

Since the groups are equally large, the possible number of mice in each group = 18/3 = 6

The number of ways of placement of mice =18!


40

For each group, the placement of mice = 6!

Hence, the required number of ways = 18!/(6!6!6!) = 18!/(6!)3

Case study
Read the Case study given below and attempt any 4 subparts:

One evening, four friends decided to play a card game Rummy. Rummy is a card game that is played with decks of
cards. To win the rummy game a player must make a valid declaration by picking and discarding cards from the two
piles given. One pile is a closed deck, where a player is unable to see the card that he is picking, while the other is an
open deck that is formed by the cards discarded by the players. To win at a rummy card game, the players have to
group cards invalid sequences and sets.

In rummy, the cards rank low to high starting with Ace, 2, 3, 4, 5, 6, 7, 8, 9, 10, Jack, Queen, and King. Ace, Jack, Queen,
and King each have 10 points. The remaining cards have a value equal to their face value. For example, 5 cards will
have 5 points, and so on. Four cards are drawn from a pack of 52 playing cards, then:

i. How many different ways can this is done

ii. Exactly one card of each suit

iii. all cards of the same suit

While reading the instructions, he observed that the question paper consists of 12 questions divided into two parts -
Part I and Part II, containing 5 and 7 questions, respectively.

Based on the given information, answer the following questions.

(i) If Anish is required to attempt 8 questions in all, selecting at least 3 from each part, then in how many ways can he
select the questions?

(ii) If Anish is required to attempt 8 questions in all, selecting at most 3 from part I, then in how many ways can he
select the questions?

Ques. Members of the BTS group, RM, Suga, V, Jin, Jungkook,Jimin, J Hope, are to be seated in a row for a photo
session

i) Total number of ways of sitting arrangement of seven members is

(a) 28 (b) 5040 (c) 2520 (d) None of these

(ii) Total number of arrangements so that RM and Jin are at extreme positions

(a) 120 (b) 60 (c) 240 (d) 360

iii) Total number of arrangements if Jungkook is sitting in the middle is

(a) 720

(b) 360

(c) 120

(d) None of these


41

(Chapter -6: PERMUTATION AND COMBINATION)


MCQ
Q -1 In how many ways 5 girls and 3 boys be seated in a row, so that no two boys are together?

5. 1440
6. 14400
7. 3600
8. None of the above

Q-2 If 2nC3 : nC2 = 12 : 1, then the value of n is ?

5. 6
6. 5
7. 4
8. None of the above

Q-3 In how many ways a committee, consisting of 5 men and 6 women can be formed from 8 men and
10 women?

5. 266
6. 5040
7. 11760
8. None of the above

Q-4 In how many different ways can the letters of the word 'GEOGRAPHY' be arranged such that the
vowels must always come together?

5. 2520
6. 2530
7. 15130
8. 15120

Q-5 In how many ways can the word CHRISTMAS be arranged so that the letters C and M are never
adjacent?

5. 8! × (7/2)
6. 9! × (7/2)
7. 8! × (9/2)
8. None of the above
Q-1 2

Q-2 2

Q-3 3

Q-4 4

Q-5 1
42

Assertion and Reason Questions


Directions: Each of these questions contains two statements, Assertion and Reason. Each of these
questions also has four alternative choices, only one of which is the correct answer. You have to select
one of the codes (a), (b), (C) and (d) given below.
(a) Assertion is correct, reason is correct; reason is a correct explanation for assertion.
(b) Assertion is correct, reason is correct; reason is not a correct explanation for assertion
(c) Assertion is correct, reason is incorrect
(d) Assertion is incorrect, reason is correct.
1. Assertion: If the letters W, I, F, E are arranged in a row in all possible ways and the words (with or
without meaning) so formed are written as in a dictionary, then the word WIFE ocurs in the 24"
position.
Reason: The number of ways of arranging four distinct objects taken all at a time is C(4,4).
2. Assertion: A number of four different digits is formed with the help of the digits 1,2,3,4,5,6,7 in all
possible ways. Then, number of ways which are exactly divisible by 4 is 200.
Reason: A number divisible by 4, if unit place digit is divisible by 4
3. Assertion: Product of five consecutive natural numbers is divisible

2 MARKS QUESTION

Question 1:

Find the 3-digit numbers that can be formed from the given digits: 1, 2, 3, 4 and 5 assuming that

a) digits can be repeated.

b) digits are not allowed to be repeated.

Solution:

a) By the multiplication principle, the number of ways in which three-digit numbers can be formed
from the given digits is 5 × 5 × 5 = 125

b) By the multiplication principle, the number of ways in which three-digit numbers can be formed
without repeating the given digits is 5 × 4 × 3 = 60

Question 2:

A coin is tossed 6 times, and the outcomes are noted. How many possible outcomes can be there?

Solution:

When we toss a coin once, the number of outcomes we get is 2 (Either Head or tail)

So, in each throw, the no. of ways to get a different face will be 2.

Therefore, by the multiplication principle, the required no. of possible outcomes is

2 x 2 x 2 x 2 × 2 × 2 = 64

Question 3:

Evaluate the following


43

(i) 6 ! (ii) 5 ! – 2 !

Solution:

(i) 6! = 1 × 2 × 3 × 4 × 5 × 6 = 720

(ii) 5! = 1 × 2 × 3 × 4 x 5 = 120

As 2! = 1 × 2 = 2

Therefore, 5 ! – 2 ! = 120-2 = 118.

Question 4:

From a team of 6 students, in how many ways can we choose a captain and vice-captain assuming
one person can not hold more than one position?

Solution:

From a team of 6 students, two students are to be chosen in such a way that one student will hold
only one position.

Here, the no. of ways of choosing a captain and vice-captain is the permutation of 6 different things
taken 2 at a time.

So, 6P2 = 6! / ( 6 -2 )! = 6! / 4! = 30

Question 5:

How many words, with or without meaning, can be formed using all the letters of the word
EQUATION, using each letter exactly once?

Solution:

Number of letters in word EQUATION` = 8

n=8

If all letters of the word used at a time

r=8

Different numbers formed = nPr

= 8P 8

= 8!/(8 8)!

= 8!/0!

= 8!/1

= 8!

=8*7*6*5*4*3*2*1

= 40320
44

3 MARKS QUESTION

1. How many numbers are there between 99 and 1000, having at least one of their digits 7?

Solution:

Numbers between 99 and 1000 are all three-digit numbers.

Total number of 3 digit numbers having at least one of their digits as 7 = (Total numbers of three-digit
numbers) – (Total number of 3 digit numbers in which 7 does not appear at all)

= (9 × 10 × 10) – (8 × 9 × 9)

= 900 – 648

= 252

2. How many 5-digit telephone numbers can be constructed using the digits 0 to 9, if each number starts
with 67 and no digit appears more than once?

Solution:

Let ABCDE be a five-digit number.

Given that the first two digits of each number are 6 and 7.

Therefore, the number is 67CDE.

As repetition is not allowed and 6 and 7 are already taken, the digits available for place C are 0, 1, 2, 3, 4, 5, 8,
9, i.e. eight possible digits.

Suppose one of them is taken at C, now the digits possible at place D is 7.

Similarly, at E, the possible digit is 6.

Therefore, the total five-digit numbers with given conditions = 8 × 7 × 6 = 336.

3. Find the number of permutations of the letters of the word ALLAHABAD.

Solution:

Given word – ALLAHABAD

Here, there are 9 objects (letters) of which there are 4As, 2 Ls and rest are all different.

Therefore, the required number of arrangements = 9!/(4! 2!)

= (1 × 2 × 3 × 4 × 5 × 6 × 7 × 8 × 9)/ (1 × 2 × 3 × 4 × 1 × 2)

= (5 × 6 × 7 × 8 × 9)/2

= 7560
45

4. In how many of the distinct permutations of the letters in MISSISSIPPI do the four Is not come
together?

Solution:

Given word – MISSISSIPPI

M–1

I–4

S–4

P–2

Number of permutations = 11!/(4! 4! 2!) = (11 × 10 × 9 × 8 × 7 × 6 × 5 × 4!)/ (4! × 24 × 2)

= 34650

We take that 4 I’s come together, and they are treated as 1 letter,

∴ Total number of letters=11 – 4 + 1 = 8

⇒ Number of permutations = 8!/(4! 2!)

= (8 × 7 × 6 × 5 × 4!)/ (4! × 2)

= 840

Therefore, the total number of permutations where four Is don’t come together = 34650 – 840 = 33810

5. In a small village, there are 87 families, of which 52 families have at most 2 children. In a rural
development programme, 20 families are to be chosen for assistance, of which at least 18 families must
have at most 2 children. In how many ways can the choice be made?

Solution:

Given,

Total number of families = 87

Number of families with at most 2 children = 52

Remaining families = 87 – 52 = 35

Also, for the rural development programme, 20 families are to be chosen for assistance, of which at least 18
families must have at most 2 children.

Thus, the following are the number of possible choices:

52C18 × 35C2 (18 families having at most 2 children and 2 selected from other types of families)

52C19 × 35C1 (19 families having at most 2 children and 1 selected from other types of families)
46

52C20 (All selected 20 families having at most 2 children)

Hence, the total number of possible choices = 52C18 × 35C2 + 52C19 × 35C1 + 52C20

5 MARKS QUESTION
1. A committee of 3 persons is to be constituted from a group of 2 men and 3 women. In how many ways
can this be done? How many of these committees would consist of 1 man and 2 women?

Solution:

Given,

Men = 2

Women = 3

A committee of 3 persons to be constituted.

Here, the order does not matter.

Therefore, we need to count combinations.

There will be as many committees as combinations of 5 different persons taken 3 at a time.

Hence, the required number of ways = 5C3

= 5!/(3! 2!)

= (5 × 4 × 3!)/(3! × 2)

= 10

Committees with 1 man and 2 women:

1 man can be selected from 2 men in 2C1 ways.

2 women can be selected from 3 women in 3C2 ways.

Therefore, the required number of committees = 2C1 × 3C2

= 2 × 3C1

=2×3

=6

2. Determine the number of 5 card combinations out of a deck of 52 cards, if there is exactly one ace in
each combination.

Solution:

Given a deck of 52 cards

There are 4 Ace cards in a deck of 52 cards.


47

According to the given, we need to select 1 Ace card out of the 4 Ace cards

∴ The number of ways to select 1 Ace from 4 Ace cards is 4C1

⇒ More 4 cards are to be selected now from 48 cards (52 cards – 4 Ace cards)

∴ The number of ways to select 4 cards from 48 cards is 48C4

Number of 5 card combinations out of a deck of 52 cards if there is exactly one ace in each combination =
4C1 × 48C4

= 4 × [48!/(44! 4!)]

= 4 × [(48 × 47 × 46 × 45 × 44!)/ (44! × 24)]

= 4 × 2 × 47 × 46 × 45

= 778320

3. A group consists of 4 girls and 7 boys. In how many ways can a team of 5 members be selected if the
team has

(i) no girls

(ii) at least one boy and one girl

(iii) at least three girls

Solution:

Given,

Number of girls = 7

Number of boys = 7

(i) No girls

Total number of ways the team can have no girls = 4C0 × 7C5

= 1 × 21

= 21

(ii) at least one boy and one girl

1 boy and 4 girls = 7C1 × 4C4 = 7 × 1 = 7

2 boys and 3 girls = 7C2 × 4C3 = 21 × 4 = 84

3 boys and 2 girls = 7C3 × 4C2 = 35 × 6 = 210

4 boys and 1 girl = 7C4 × 4C1 = 35 × 4 = 140


48

Total number of ways the team can have at least one boy and one girl = 7 + 84 + 210 + 140

= 441

(iii) At least three girls

Total number of ways the team can have at least three girls = 4C3 × 7C2 + 4C4 × 7C1

= 4 × 21 + 7

= 84 + 7

= 91

9. How many numbers greater than 1000000 can be formed using the digits 1, 2, 0, 2, 4, 2, 4?

Solution:

Given numbers – 1000000

Number of digits = 7

The numbers have to be greater than 1000000, so they can begin either with 1, 2 or 4.

When 1 is fixed at the extreme left position, the remaining digits to be rearranged will be 0, 2, 2, 2, 4, 4, in
which there are 3, 2s and 2, 4s.

Thus, the number of numbers beginning with 1 = 6!/(3! 2!) = (6 × 5 × 4 × 3!)/(3! × 2)

= 60

The total numbers begin with 2 = 6!/(2! 2!) = 720/4 = 180

Similarly, the total numbers beginning with 4 = 6!/3! = 720/6 = 120

Therefore, the required number of numbers = 60 + 180 + 120 = 360.

10. 18 mice were placed in two experimental groups and one control group, with all groups equally large. In
how many ways can the mice be placed into three groups?

Solution:

Number of mice = 18

Number of groups = 3

Since the groups are equally large, the possible number of mice in each group = 18/3 = 6

The number of ways of placement of mice =18!

For each group, the placement of mice = 6!

Hence, the required number of ways = 18!/(6!6!6!) = 18!/(6!)3


49

Case study
Read the Case study given below and attempt any 4 subparts:

One evening, four friends decided to play a card game Rummy. Rummy is a card game that is played with decks of
cards. To win the rummy game a player must make a valid declaration by picking and discarding cards from the two
piles given. One pile is a closed deck, where a player is unable to see the card that he is picking, while the other is an
open deck that is formed by the cards discarded by the players. To win at a rummy card game, the players have to
group cards invalid sequences and sets.

In rummy, the cards rank low to high starting with Ace, 2, 3, 4, 5, 6, 7, 8, 9, 10, Jack, Queen, and King. Ace, Jack, Queen,
and King each have 10 points. The remaining cards have a value equal to their face value. For example, 5 cards will
have 5 points, and so on. Four cards are drawn from a pack of 52 playing cards, then:

i. How many different ways can this is done

ii. Exactly one card of each suit

iii. all cards of the same suit

While reading the instructions, he observed that the question paper consists of 12 questions divided into two parts -
Part I and Part II, containing 5 and 7 questions, respectively.

Based on the given information, answer the following questions.

(i) If Anish is required to attempt 8 questions in all, selecting at least 3 from each part, then in how many ways can he
select the questions?

(ii) If Anish is required to attempt 8 questions in all, selecting at most 3 from part I, then in how many ways can he
select the questions?

Ques. Members of the BTS group, RM, Suga, V, Jin, Jungkook,Jimin, J Hope, are to be seated in a row for a photo
session

i) Total number of ways of sitting arrangement of seven members is

(a) 28

(b) 5040

(c) 2520

(d) None of these

(ii) Total number of arrangements so that RM and Jin are at extreme positions

(a) 120 (b) 60 (c) 240 (d) 360

iii) Total number of arrangements if Jungkook is sitting in the middle is

(a) 720 (b) 360 (c) 120 (d) None of these


50

(Chapter -7: BINOMIAL THEOPREM)


MCQ {5 QUESTIONS}

11) The number of terms in ((𝑥 − 5)2 )5I


A) 8 terms B) 9 terms C) 10 terms D)11 terms
2 Which one is True?
A) (1.2)4000 > 800 B) (1.2)4000 < 800
C) (1.2)4000 = 800 D)(1.2)4000 = 1600
3 1 9
The term independent of x in the expansion of (2𝑥 + 3𝑥 2 ) is
A) 2nd B) 3rd C) 4th D) 5th
4 𝑥 2 6
In the expansion of (3 − 𝑥) , 𝑥 > 0, the constant term is
160 16 −160 120
A) 27 B)27 C) 27 D) 27
5 If the coefficient of 𝑥 2 and 𝑥 3 in the expansion of (3 + 𝑚𝑥)9 are equal , then the value of m is
9 7 9 7
A) − 7 B) − 9 C) 7 D) 9

ASSERTION-REASON BASED QUESTIONS {2 QUESTIONS}

In the following questions, a statement of Assertion (A) is followed by a statement of Reason (R). Choose the
correct answer out of the following choices.
(a) Both (A) and (R) are true and (R) is the correct explanation of (A).
(b) Both (A) and (R) are true but (R) is not the correct explanation of (A).
(c) (A) is true but (R) is false.
(d) (A) is false but (R) is true.

Q1: Assertion (A) : The expansion of (1+ x)n = nC0 + nC1 x + nC2 x2+…+ nCn xn
Reason (R) : If x = −1, then the above expansion is zero
2
Q2: Assertion (A) : ∑𝑛 𝑘=1 𝑘(𝑛𝑐 𝑛 ) = n (2𝑛 − 1𝑐 𝑛−1 )
Reason (R) : : If 22003 is divided by 15 then remainder is 8.
{2 marks questions}
Q1: Using binomial theorem, evaluate (99)5 .
3 4
Q2: Expand (x 2 + x) , x ≠ 0 b using Pascal triangle.
Q3: Expand (1 + 𝑥 + 𝑥 2 )4
Q4: Prove that(𝑎 + 𝑏)4 − (𝑎 − 𝑏)4 is an even number if a and b are integers:
3 6
Q5: Find 4th term of the expansion (2𝑥 + 𝑥)
{3 marks questions}
Q1: Prove that ∑𝑛𝑟=0 3𝑟 𝑛 𝐶𝑟 = 4𝑛
Q2: Find a, if the 4th and 5th term of the expansion (2 + a)7 are equal.
Q3: Which of the following is larger (9950 + 10050 ) or 10150
1 6
Q4: Find the term independent of x in the expansion of(2x + 3x2 ) .
6 6
Q5:Find (a + b)6 − (a − b)6 hence evaluate(√3 + √2) − (√3 − √2)
{5 marks questions}
51

4 4
Q1: Find the value of (a2 + √a2 − 1) + (a2 − √a2 − 1)
Q2: Prove that 12n − 11n − 1 is divisible by 121 For all n∈N by using binomial theorem.
Q3: Show that 24𝑛+4 − 15𝑛 − 16, where n ∈ N is divisible by 225.

CASE STUDY BASED QUESTIONS {3 QUESTIONS}

Q1: In class XI, teacher explained binomial theorem. Two students Shivani and Vishwani trying to solve the
exercise. Shivani expanded (1+x)6 by using Binomial theorem, Viswani expanded (x+1)6.
Based on this above information answer the following questions.
1. According to Shivani find 4th term .
2. Find the value of 6C0 + 6C1 + 6C2 + 6C3 + … … + 6C6

3. Find the positive value of x if 3rd terms of Shivani and Vishwani are equal.
Q2: In class XI, teacher explained binomial theorem. Two students
Rohan and Mahek trying to solve the exercise. Mahek expanded
(x+y)3 by using Binomial theorem, Rohan expanded (x+y)3.by
using algebraic identity
Based on this above information answer the following
questions.
1. According to Mahek find expansion
2. According to Rohan find expansion
3. Find the value of (99)3 by using above result

(Chapter -8: SEQUENCE AND SERIES)


MCQS
Q1 First five terms of the sequence 𝑎𝑛 =(−1)𝑛−1 5𝑛+1 are
(a)25,125,625,-3125,15625
(b)25,-125,,625,-3125,15625
(c)25,125,625,-3125,-15625
(d)-25,125,-625,3125,-15625
Q2How many multiples of 4 lie between10 and 250
(a)126
(b) 23
(c)60
(d)130
Q3 If n Arithmetic means are introduced between 3 and 17 such that ratio of last mean to first mean is 3:1,
then value of n is
52

(a)6
(b)8
(c)4
(d)None of these
Q4 If the sum of first n even natural numbers is equal to k times the sum of first n odd natural numbers ,then k
is
1
a) 𝑛
𝑛−1
(b) 𝑛

𝑛+1
(c) 2𝑛

𝑛+1
(d) 𝑛

Q5If a,b,c are in AP and x,y,z are in GP , then value of 𝑥 𝑏−𝑐 𝑦 𝑐−𝑎 𝑧 𝑎−𝑏 is
(a)0
(b)1
(c)xyz

(d) 𝑥 𝑎 𝑦 𝑏 𝑧 𝑐

ASSERTION AND REASONING QUESTIONS


3 4
Q 1 ASSERTION: If the sum of the series + +√5 +……….. to 25 terms is 75√5
√5 √5

REASON : If 27,x,3 are in GP, then x=±4

(a) Both A and R are true and R is the correct explanation of A.


(b) Both A and R are true and R is not the correct explanation of A.
(c) A is true but R is false.
(d) Both A and R false.
−2 −7
Q 2 ASSERTION: If the numbers , K, 2 are in GP ,then K=±1
7
𝑎 ,𝑎
REASON : If 𝑎1, 𝑎2 , 𝑎3 are in GP, then 𝑎 2 = 𝑎 3
1, 2

(a)Both A and R are true and R is the correct explanation of A.


(b)Both A and R are true and R is not the correct explanation of A.
©A is true but R is false.
(d)Both A and R false.
53

VERY SHORT ANSWERS ( 2 MARKS)


Q 1 Insert two arithmetic means between 2 and 10.
Q 2 Insert a geometric mean between 0.008 and 0.2 .
2𝑛 −1
Q 3 The sum of n terms of a progression is , show that it is a GP .
3

Q 4 Find 𝑎8 of the sequence 0.3,0.06, 0.012,…………..

Q 5 Find the sum of two arithmetic means between 6 and 27.

SHORT ANSWERS ( 3 MARKS )


Q 1 The ratio of the sum of first three terms is to that of first 6 terms of a GP is 125 : 152 .Find the common
ratio
1 1 1
Q 2 If a, b ,c are in GP , and 𝑎𝑥 =𝑏 𝑦 =𝑐 𝑧 ,prove that x,y ,z are in AP.
Q 3 The sum of four numbers in GP is 60 and the arithmetic mean of the first and the last is 18 .Find the
numbers.
𝑎𝑛 +𝑏 𝑛
Q 4 If 𝑎𝑛−1 +𝑏𝑛−1 is the AM between a and b ,then find value of n.

Q 5 Find the sum of the series up to n terms

0.5+0.55+0.555+……………..

LONG ANSWERS (5 MARKS)


Q1. If the A.M. between pth and qth terms of an A.P. be equal to the A.M. between the rth and sth terms of
A.P., then show that p+q = r+s.

Q2. If n arithmetic means are inserted between 20 and 80 such that the ratio of first mean to the last mean is
1:3, then find the value of n.

Q3. On the first day of new year i.e., on January Ramesh helped 3 persons. When those persons thanked him,
he advised them not to thank but to help 3 more persons on second day and instructed them to do the same on
third day. They move the chain similarly.
Assuming that the chain is not broken, answer the following:
(i) Find how many persons will be helped on 5th day.
(ii) Find the total number of people helped in 5 days.
(iii) 6561 persons will be helped on which day?

OR
(iii)How many persons will be helped on 7th day?

SOLUTIONS (MCQ)

ANS1 (b)25,-125,,625,-3125,15625
54

ANS 2 (c)60

ANS 3 (a)6
𝑛+1
ANS 4 (c) 2𝑛

ANS 5 (b)1

(ASSERTION AND REASONING QUESTION)

ANS 1 © A is true but R is false.

ANS 2 (a)Both A and R are true and R is the correct explanation of A.


(VERY SHORT ANSWERS)

ANS -1
14
a=2 ,𝑎4 = 10 ,2+3d = 10 d=8/3 , 𝑎2 = , 𝑎3 =22/3
3

ANS-2
G= 0.04
ANS -3 ,
2𝑛 −2𝑛−1
𝑎𝑛 =
3
𝑎2 𝑎
= 𝑎3 =2
𝑎1 2

ANS 4
a=0.3 , r= 0.2

𝑎8 =0.3(0.2)7
ANS 5

a=6 ,𝑎4 =27, 6+3d =27 , d=7 , 𝑎2 = 13 𝑎3=20 ,sum=33

( SHORT ANSWERS)
a(𝑟 3 −1) 125
ANS 1 𝑎(𝑟 6 −1)=152

1 125
=152
𝑟 3 +1

r=3
1 1 1
ANS 2 Let 𝑎𝑥 =𝑏 𝑦 =𝑐 𝑧 = 𝑘, a=𝑘 𝑥 ,b=𝑘 𝑦 ,c=𝑘 𝑧

𝑏 2 =ac, 𝑘 2𝑦 =𝑘 𝑥+𝑧
2y=x+z
55

X,y,z are in AP
ANS 3

a+ar+𝑎𝑟 2+𝑎𝑟 3=60

a(1+r)(1+𝑟 2 )=60 ……..(1)

a+𝑎𝑟 3=2x18

a(1+𝑟 3 )=36, a(1+r)(1+𝑟 2 -r) =36 …….(2)


dividing (2) by (1)

2𝑟 2 -5r+2=0
1
r =2, 2 , a=32

numbers are 32,16,8,4


ANS 4
𝒂𝒏 +𝒃𝒏 𝒂+𝒃
=
𝒂𝒏−𝟏 +𝒃𝒏−𝟏 𝟐

By cross multiplication

𝑎𝑛−1 (a-b)=𝑏 𝑛−1 (𝑎 − 𝑏)

𝑎𝑛−1 =𝑏 𝑛−1
𝑎𝑛−1
=1
𝑏 𝑛−1
𝑎 𝑛−1 𝑎
(𝑏) =(𝑏 ) 0

n-1=0

n=1

ANS 5

𝑆𝑛 = 0.5 +0.55 +0.555+……….


5
= 9(0.9+0.99+0.999+………)
5
=9((1-0.1)+(1-0.01)+(1-0.001)+………)
5
= 9 [ (1 + 1 + ⋯ 𝑛 𝑡𝑖𝑚𝑒𝑠) − (0.1+0.01+0.001+………up to nth term)

5 0.1(1−0.1𝑛 )
= 9 [ n- ]
1−0.1

5 1(1−0.1𝑛 )
= 9 [ n- ]
9

5
= 81 [ 9 n- 1+0.1𝑛 ]

LONG ANSWERS SOLUTION (5 MARKS)


56

ANS 1) Let a be the first term and d be the common difference of the given A.P. then
Ap = pth term = a + ( p – 1 ) d ; aq = qth term = a + ( q - 1 ) d
Ar = rth term = a + (r - 1) d and, as = sth term = a + (s -1) d
It is given that
A.M. between ap and aq = A.M. between ar and as
1 1
=> (ap + aq) = 2 (ar + as)
2

=> ap + aq = ar + as
=> {a + (p - 1) d} + { a + (q - 1) d} = {a + (r -1) d} + {a + (s-1) d}
=> (p + q - 2) d = (r + s - 2) d
=> p + q = r + s

ANS 1 We have A,𝑃1 , 𝑃2 , 𝑃3 ,……… 𝑃14 ,B

(i) B is 14+2 = 16 th term.


(ii) 𝑎16 =3 ,a=1, 1+15d=3 ,d=2/15
(iii) D is the distance between two consecutive pillars
(iv) Distance of third pillar =𝑎4 =1+3(2/15) =7/5km
OR
Position of 14th pillar =𝑎15 =43/15 km
ANS 2
(i) Let side be x
𝑥 2 =502 +502 , x= 50√2 cm
(ii) Let side be y
𝑦 2 =(25√2)2+(25√2)2 , y= 50 cm
𝑝𝑒𝑟𝑖𝑚𝑒𝑡𝑒𝑟= 4y =4x50 =200 cm

(iii) Length of each side Ist square, IIsquare ,……. Form a GP 100, 50√2,50….
1 25
Side of 6th square = 𝑎6 =100( )5=
√2 √2
OR
(𝑖𝑖𝑖)𝑎7=12.5
ANS 3
(i) No of persons helped by others form a GP
3,9,27,…………….

𝑎5 =3(3)4 =243
(𝑖𝑖)toal no of people got help on 5th day =3+9+27+81+243=363
(𝑖𝑖𝑖)8th day
𝑂𝑅
(iii) 𝑎7 =2187
57

(Chapter -9: STRAIGHT LINES)


MULTIPLE CHOICE QUESTIONS
1) The equation of straight line parallel to the 𝑥 − 𝑎𝑥𝑖𝑠 at a distance 5 units below it is
(a) 𝑦 = −5 (b) 𝑦 = 5 (c) 𝑥 = −5 (d) 𝑥 = 5
2) The inclination of the line √3 𝑥 − 𝑦 − 1 = 0 is
(a) Ɵ = 300 (b) Ɵ =600 (c) Ɵ = 450 (d) Ɵ = 900
3) The distance between the lines 3𝑥 + 4𝑦 = 9 and 6𝑥 + 8𝑦 = 15 is
(a) 6 (b) 3 (c) 1/3 (d) 3/10
4) The point on y-axis which is equidistant from the points ( 3,2 ) and ( - 5, - 2 ) is
(a) ( - 2,0 ) (b) ( 0, - 2 ) (c) ( 0, - 1 ) (d) ( - 1,0 )
5) Line through the points ( - 2,6 ) and ( 4,8 ) is perpendicular to the line through the points (8,12 ) and
( x,24 ) . The value of x is
(a) 4 (b) 3 (c) 2 (d) 1

ASSERTION - REASONING
DIRECTION: In the following questions, a statement of assertion (A) is followed by a statement of reason
(R). Mark the correct choice as:
(a) Both assertion (A) and reason (R) are true and reason (R) is the correct explanation of assertion (A).
(b) Both assertion (A) and reason (R) are true but reason (R) is not the correct explanation of assertion (A).
(c) Assertion (A) is true but reason (R) is false.
(d) Assertion (A) is false but reason (R) is true.
1) Assertion : The equation of the straight line which passes through the point ( 2, - 3) and the point of the
intersection of the lines x + y + 4 = 0 and 3x – y – 8 = 0 is 2x – y – 7 = 0
Reason : Product of slopes of two perpendicular straight lines is - 1 .
2) Assertion : Equation of the horizontal line having distance a from the x-axis is either y = a or y = - a
Reason : Equation of the vertical line having distance b from the y-axis is either x = b or x = - b .

VSA type QUESTIONS ( 2 Marks )


1) Find the distance between the parallel lines 15𝑥 + 8𝑦 − 34 = 0 and 15𝑥 + 8𝑦 + 31 = 0 .
2) Find the slope of the line which passes through the origin and the mid point of the line segment joining
the points (0, - 4) and (8,0) .
3) Find the value of k for which the line (𝑘 − 3)𝑥 − (4 − 𝑘 2 )𝑦 + 𝑘 2 − 7𝑘 + 6 = 0 is parallel to
x-axis .
4) Three consecutive vertices of a rhombus are (5,3) , (2,7) and (- 2,4) . Find the fourth vertex .
5) Find the equation of straight line through (2,3) and making an angle of 600 with the positive direction
of x-axis .
SA type QUESTIONS ( 3 Marks )
1) Find the coordinates of the foot of the perpendicular drawn from the point (- 1,3) to the line 3𝑥 −
4𝑦 − 16 = 0 .
2) In what ratio the line joining the points A (- 1,1) and B (5,7) is divided by 𝑥 + 𝑦 = 4 .
58
𝑥 𝑦
3) If p is the length of the perpendicular from origin to the line 𝑎 + = 1 , then prove that
𝑏
1 1 1
= 𝑎2 + 𝑏 2
𝑝2
4) At what point the origin be shifted , if the coordinates of a point (4,5) become ( - 3,9) ?
5) Find the equation of a line perpendicular to the line 𝑥 − 7𝑦 + 5 = 0 and having x-intercept 3.Also find
the distance of the point (1, - 1) from the required line .
LA type QUESTIONS ( 5 Marks )
1) Find equations of lines which pass through the point ( 3,4 ) and the sum of its intercepts on the axes is
14 .
2) (i) Find the equation of the line which intersects x-axis at a distance 2 units in the right of origin and
y-axis at distance 3 units below to the origin.
(ii) Find the distance of point (-2 , 3) from the line 5𝑦 = 12𝑥 − 2 .
(iii) Find the angle between the x-axis and the line joining the points (3, -1) and (4, -2)

CASE STUDY -1
A girl standing at the junction (crossing) of two straight paths represented by the equations 2x –
3y + 4 = 0 and 3x + 4y – 5 = 0 wants to reach the path whose equation is 6x – 7y + 8 = 0 in the
least time.

(i) Equation of path that she should follow is


a) 119 x+ 102 y = 125
b) 109 x+ 102 y = 125
c) 119 x+ 112 y = 125
d) 109 x+ 102 y = 105

(ii) The angle between the cross roads is


a) tan-1 (6/17)
b) tan-1 (1/17)
c) tan-1 (17/6)
d) tan-1 (6)

(iii) The y- intercept of the path used to reach in least time at 6x – 7y + 8 = 0 is


a) 125 / 119
b) 119 / 125
c) 102 / 125
d) 125/ 102
(iv) The x – intercept of the path used to reach in least time at 6x – 7y + 8 = 0 is
a) 125 / 119
59

b) 119 / 125
c) 102 / 125
d) 125/ 102

CASE STUDY -2
A triangular park has two of its vertices as B(-4, 1) and C(2, 11). The third vertex A is a point dividing the
line joining the points (3, 1) and (6, 4) in the ratio 2 : 1.
Based on the above information, answer the following questions:
a) The coordinates of third vertex A are
i) (5, 3)
ii) (3, 5)
iii) (-5, 3)
iv) (5, -3)
b) The equation of passing through B and C is
i) 5x – 3y – 23 = 0
ii) 5x – 3y + 23 = 0
iii) 3x + 5y – 23 = 0
iv) 5x + 3y – 23 = 0

c) The equation of line passing through A and parallel to BC is


i) 5x – 3y + 16 = 0
ii) 5x – 3y + 34 = 0
iii) 5x – 3y – 16 = 0
iv) 5x + 3y – 16 = 0
d) The equation of line passing through A and perpendicular to BC is
i) 3x + 5y – 30 = 0
ii) 3x + 5y + 30 = 0
iii) 3x – 5y + 30 = 0
iv) 3x + 5y = 0
e) The area of triangular field ABC is
i) 78 sq units
ii) 43 sq units
iii) 86 sq units
iv) 39 sq units

CASE STUDY -3
60

Read the paragraph given below and answer the following queations:
Villages of Shanu and Arun’s are 50 km apart and are situated on Delhi Agra highway as shown in
figure .
Another highway yy′ crosses Agra Delhi highway at O (0,0). A small local road PQ crosses both
the highways at points A and B s.t. OA = 10 km and OB = 12 km. Also, the villages of Barun and
Jeetu is 15km from O

Now answer the following questions:


a) What are the coordinates of A?
i) (10, 0)
ii) (10, 12)
iii) (0, 10)
iv) (0, 15)
b) What is the equation of line AB?
i) 5x + 6y = 60
ii) 6x + 5y = 60
iii) x = 10
iv) y = 12
c) What is the distance of AB from O (0,0)?
i) 60 km
ii) 60/√61 km
iii) √61 km
iv) 60 𝑘𝑚
d) What is the slope of line AB?
i) 6/5
ii) 5/6
iii) -6/5
iv) 10/12
e) What is the length of line AB?
61

i) √61 km
ii) 12 𝑘𝑚
iii) 10 km
iv) 2√61 𝑘m

ANSWERS
MCQ
1) (a) 𝑦 = −5
2) (b) Ɵ = 600
3) (d) 3/10
4) (b) ( 0, - 2)
5) (a) 4
ASSERTION AND REASON
1) (b)
2) (b)

VSA type QUESTIONS ( 2 Marks )


|𝑐 − 𝑑|
1) Distance between lines 𝑎𝑥 + 𝑏𝑦 + 𝑐 = 0 𝑎𝑛𝑑 𝑎𝑥 + 𝑏𝑦 + 𝑑 = 0 is √𝑎2 + 𝑏 2
|−34−31|
Distance = ……………………………………………………… 1
√225+64
|− 65 | 65
= = 17 ……………………………………………………... 1
√289

0+8 − 4+0
2) Mid point of line segment joining (0, - 4) and (8,0) = ( , ) = (4, - 2) ……..1
2 2
− 2−0 1
Slope of required line = = -2 ……………………………………… 1
4−0

3) Since line is parallel to x-axis slope of the line = 0 ……………………………..1/2


−(𝑘−3)
= 0 ……………………………….1/2
−(4−𝑘 2 )

𝑘 − 3 = 0 ⟹ 𝑘 = 3 ………………….. 1
4) Let (a,b) be the fourth vertex
In rhombus diagonals bisect each other. ……………………………. 1/2
5+ −2 3+4 2+𝑎 7+𝑏
⟹( 2 , 2 )=( 2 , 2 ) ………………………………………. 1/2
⟹ a =1 , b = 0 ……………………………………….. 1

5) Slope m = tan 600 = √3 ………………………………. 1


Equation of line is 𝑦 − 3 = √3 (𝑥 − 2)
⟹ √3𝑥 − 𝑦 + (3 − 2√3 ) = 0 ……………………………… 1

SA type QUESTIONS ( 3 Marks )


1) Let P (- 1,3) be the given point
62

M(a,b) be the foot of the perperdicular on the line


𝑏−3
Slope of the line joining (- 1,3) and (a,b) = 𝑚1 = 𝑎+1
3 3
Slope of line 3𝑥 − 4𝑦 − 16 = 0 ⟹ 𝑦 = 4 𝑥 − 4 ⟹ 𝑚2 = 4 ………………… 1
The two lines are perpendicular , 𝑚1 𝑚2 = - 1
𝑏−3 3
(𝑎+1) 𝑋 (4) = −1 ⟹ 3𝑏 − 9 = −4𝑎 − 4 ⟹ 4𝑎 + 3𝑏 = 5
Point (a,b) lies on 3𝑥 − 4𝑦 − 16 = 0 ⟹3𝑎 − 4𝑏 = 16 ………………………… 1
68 49
Solving 𝑎 = 25 and 𝑏 = − 25

68 49
Foot of perpendicular = (25 , − 25) ………………………… 1

2) Let k :1 be the ratio in which the line joining A (- 1,1) and B (5,7) is divided by 𝑥 + 𝑦 = 4
5𝑘−1 7𝑘+1
The point of intersection = ( 𝑘+1 , 𝑘+1 ) …………………… 1
5𝑘−1 7𝑘+1
The point also lie on the line 𝑥 + 𝑦 − 4 = 0 ⟹ 𝑘+1 + 𝑘+1 − 4 = 0 ……….. 1
⟹ 8𝑘 − 4 = 0
1
⟹ 𝑘 = 2 ⟹ Ratio = 1 : 2 ……….. 1

𝑥 𝑦
3) Length of perpendicular from (0,0) to 𝑎 +𝑏 = 1 is p

0+0−1
⟹ | 1 1
|=p ……………………………………….. 1
√ 2+ 2
𝑎 𝑏
1
⟹ 1 1
=p …………………………………………. 1
√ 2+ 2
𝑎 𝑏

1 1 1
⟹ √𝑎2 + 𝑏2 = 𝑝
1 1 1
⟹ + 𝑏 2 = 𝑝2 …………………………………………. 1
𝑎2

4) If (h,k) is the new origin ,then new coordinates (X,Y) of the point (x,y) are given by 𝑥 = 𝑋 + ℎ and
𝑦 =𝑌+𝑘 ……………………………… 1
(x,y) = (4,5) and (X , Y) = ( -3,9)
⟹ 4 = - 3 + h and 5 = 9 + k ……………………………… 1
⟹ h = 7 and k = - 4
Origin = (7, - 4) ………………………………. 1

5) Given equation 𝑥 − 7𝑦 + 5 = 0
1 5
⟹ 𝑦 = 7𝑥 + 7
1
Slope of given line = 7 …………………………… 1
Slope of line perpendicular to it is - 7
Required line has x-intercept = 3, so it passes through (3,0)
Equation of required line 𝑦 − 0 = −7(𝑥 − 3) ⟹ 7𝑥 + 𝑦 − 21 = 0 ……… 1
|7𝑋1+(−1)−21|
Distance of the point (1, - 1) from required line = √72 +12
|−15| 15 3
= = = ………. 1
√50 5√2 √2

LA type QUESTIONS ( 5 Marks )


1) x intercept a = a
y intercept b = 14 – a ……………. 1
63
𝑥 𝑦 𝑥 𝑦
Equation of line ; + =1 ⟹𝑎+ =1 ……………….. 1
𝑎 𝑏 14−𝑎
𝑥 𝑦 3 4
Line 𝑎 + 14−𝑎 = 1 is passing through (3,4) ⟹ 𝑎 + 14−𝑎 = 1
Simplifying as 𝑎2 − 13 𝑎 + 42 = 0 ………………… 1
Solving above equation ; 𝑎 = 6 ,7 ⟹ 𝑎 = 6, 𝑏 = 8 𝑜𝑟 𝑎 = 7, 𝑏 = 7 ………… 1
Getting equation of lines ; 4𝑥 + 3𝑦 = 24 , 𝑥 + 𝑦 = 7 …………. 1

2) (i) x-intercept ; a = 2
y-intercept ; b = -3 …………………………. 1
𝑥 𝑦 𝑥 𝑦
Equation of line ; 𝑎 + 𝑏 = 1 ⟹ + =1
2 −3
Simplifying 3𝑥 − 2𝑦 − 6 = 0 ……………………… 1
( 12𝑋−2 )−(5 𝑋 3)−2 41
(ii) Distance = ⌈ 2 2
⌉= ……………………… 1
√12 +(−5) 13
(iii) Slope of x-axis ; m1= 0 ………………….. 1/2
Slope of line joining the points (3, -1) and (4, -2) ; m2 = -1 ……….. 1/2
Calculation of angle = 450 …………………….. 1

CASE STUDY -1
(i) A (ii) C (iii) D (iv) A

CASE STUDY -2

a) i) (5, 3)
b) ii) 5x – 3y + 23 = 0
c) iii) 5x – 3y – 16 = 0
d) i) 3x + 5y – 30 = 0
e) iv) 39 sq units

CASE STUDY -3

a) i) (10, 0)
b) ii) 6x + 5y = 60
c) ii) 60/√61 km
d) iii) -6/5

e) iv) 2√61 𝑘m
64

(Chapter -10 : CONIC SECTION)

1. The equation of the hyperbola with vertices at (0,±6) and e=5/3 is


𝑥2 𝑦2 𝑦2 𝑥2
(a) – =1 (b) – =1
36 64 36 64

𝑥2 𝑦2 𝑦2 𝑥2
(c) – =1 (d) – =1
64 36 36 64
𝑦2 𝑥2
Ans ( b) Since the vertices are on the y-axis ,the equation is of the form (a) –𝑏2 =1 as vertices are (0,±6) ,
𝑎2
𝑦2 𝑥2
a=6 so the required equation is –64 =1
36

2. The conic represented by x=2(cos t + sin t) ,y=5(cos t - sin t) is


(a) a circle (b) a parabola (c) an Ellipse (d) a hyperbola

Ans From given equation x/2 =(cos t + sin t) ,y/5 =(cos t - sin t) Elliminating t from i & ii, we have
𝑥2 𝑦2
+25 =2 is an ellipse
4

3. Equation of the circle with centre on the y-axis and passing through the origin and the point (2, 3) is
2 2 2 2
(a) x + y – 13y = 0 (b) 3x + 3y +13x + 3 = 0
2 2 2 2
(c) 6x + 6y – 13x = 0 (d) x + y + 13x + 3 = 0

Ans
65

4. The foci of an ellipse are (0, ± 6) and the length of its minor axis is 16. The equation of the ellipse is

Ans

2
5. If the parabola x = 6y, the focus and the equation of directrix are respectively.

2
Ans: (c), given equation of the parabola is x = 4ay ⇒ a = 3/2 Focus of the parabola is Its directrix is y = – a ⇒ y
= -3/2

Assertion & Reasoning Question


Each of these questions contains two statements Assertion & Reason. Each of these questions also has four alternatives
choices,

a) Assertion is correct. Reason is correct, Reason is correct explanation for assertion


b) Assertion is correct. Reason is correct, Reason is not a correct explanation for assertion
c) Assertion is correct. Reason is incorrect,.
d) Assertion is incorrect. Reason is correct,

1.Assertion: Latus rectum of a parabola ia a line segment perpendicular to the axis of the parabola,through the focus and
whose end points lie on the parabola.

𝑥2 𝑦2
Reason : The equation of the hyperbola with foci on the y-axis is – =1
𝑎2 𝑏2
Ans: c) Assertion is correct but Reason is incorrect
66

2. Parabola is symmetric with respect to the axis of the parabola.

Assertion: If the equation has a term y2,then the axis of symmetry is along the x-axis,

Reason: If the equation has a term x2,then the axis of symmetry is along the x-axis

Ans (c)

3. Let the centre of an Ellipse is at (0,0)


Assertion:If major axis is on the y-axis and ellipse passes through the points (3,2) and (1,6), then the equation of ellipse
is x2/10+y2/40=1

Reason: x2/a2 + y2/ b2=1 is an equation of ellipse if major axis is along y-axis

Ans

Question carry 2 marks each:


1.Find the length of the axes, coordinates of foci, eccentricity of the conic section 16x2 – 9y2 – 144 = 0.

ANS: Given conic section is


16x2 – 9y2 – 144 = 0
⇒ 16x2 – 9y2 = 144

⇒ =1 [Hyperbola]
Here a2 = 9 ⇒a = 3 and b2 = 16 ⇒b = 4

Eccentricity (e) =
Length of transverse axis = 2a = 6
Length of conjugate axis = 2b = 8

Foci = (± ae, 0) = = (± 5, 0)
67

2. Find the coordinates of the focus, the equation of directrix, vertex and length of latus rectum for the parabola y2 = –
12x.

ANS: Given parabola is y2 = – 12x


⇒ 4a = – 12 ⇒a = – 3
∴ Focus is (–3, 0);
Directrix is x + a = 0 ⇒x – 3 = 0
∴ Vertex is (0, 0);
Length of latus rectum = 4a = 12

3. What is the eccentricity of hyperbola whose vertices and foci are (± 2, 0) and (± 3, 0) respectively ?

ANS: Vertices are (± 2, 0), foci are (± 3, 0)


⇒a = 2, c = 3

∴e = .

4. Find the centre and radius of the circle 2x2 + 2y2 – x = 0.

Ans Centre =

And radius =1/4


5. For the following curve, find the coordinates of the focus, axis, equations of the directrix and length of the
latusrectum

:x2 = – 16y.
2
ANS: Consider the equation x = – 16y
Comparing with x2 = 4ay, we get
4a = – 16 ⇒a = – 4.
∴ Coordinates of focus are (0, a), i.e. (0, – 4)
Axis is y-axis.
Equation of directrix is y + a = 0 ⇒y – 4 = 0
Length of latus rectum = | 4a | = 16.

Question carry 3 marks each


1.Show that 4x2 + y2 – 8x + 2y + 1 = 0 represents an ellipse. Find its eccentricity and length of latus rectum,

ANS: Consider the equation,


4x2 + y2 – 8x + 2y + 1 = 0
⇒ 4x2 – 8x + y2 + 2y + 1 = 0
⇒ 4[x2 – 2x] + [y2 + 2y + 1] = 0
2 2
⇒ 4[(x – 1) –1)] + (y + 1) = 0
⇒ 4(x – 1)2 – 4 + (y + 1)2 = 0
2 2
⇒ 4(x – 1) + (y + 1) = 4

⇒ represents an ellipse.
Let X = x – 1, Y = y + 1 ... (i)
Also a2 = 4 ⇒a = 2 ; b2 = 1 ⇒b = 1

Eccentricity =

Length of Latus rectum =


68

2. Find the equation of the hyperbola whose foci are (± 3√ 5 , 0) and the length of the latus rectum is 8.

Ans Let hyperbola be ...(i)


Foci are (±3√ 5, 0) and latus rectum is 8.

∴c = 3√ 5 and =8
2
⇒ and b = 4a
2 2 2
⇒a + b = 45 and b = 4a ...(ii)
2
⇒a + 4a – 45 = 0
⇒ (a + 9) (a – 5) = 0
⇒a = – 9, a = 5
a = – 9, not possible [From (ii)]
∴a = 5
Substituting in (i), we get
2
b = 4 × 5 = 20

Substituting for a, b in (i), we get hyperbola as .

3. Write the equation of ellipse whose vertices are (± 5, 0) and foci (± 4, 0).

ANS: Vertices are (± 5, 0), foci (± 4, 0)

Ellipse is …(i)
We have a = 5, c = 4 =
2 2
⇒ 16 = 25 – b ⇒b = 9
Substituting for a, b in (i), we get

= 1 as equation of the ellipse.

4. Find the equation of the circle passing through (0, 0) and making intercepts a andb on the coordinate axes.

ANS: Circle passes through origin. Let it meets axes at A, B. Then AB is

Coordinates of centre are

and radius is

∴ Equation of circle is
2 2
⇒x + y – ax – by = 0 is the required equation.

5. Find the equation of a circle whose centre is (3, –2) and which passes through the inter-section of the lines 5x + 7y = 3
and 2x –3y = 7.

ANS: Point of intersection of 5x + 7y = 3


and 2x – 3y = 7 is (2, – 1)
Centre is (3, – 2).

Therefore, r =
69
2 2
∴ Circle is (x – 3) + (y + 2) = 2
2 2
x + y – 6x + 4y + 11 = 0

CBQ Question carry 5 marks each


1. A man is running on a race course such that the sum of distances of two flags posts from him is always 26 m and
distance between two flag post is 10 m.

(i) The equation of the path is

(a) (b)

(c) (d)

(ii) The eccentricity of the path is

(a) (b)

(c) (d)

(iii) The length of latus rectum of the curve is

(a) (b)

(c) (d)

(iv) Coordinates of vertices of the path are

(a) (0, ± 12) (b) (± 13, 0)

(c) (0, ± 16) (d) (± 5, 0)

(v) Coordinates of the foci of the path are

(a) (± 5, 0) (b) (± 4, 0)

(c) (0, ± 13) (d) (0, ± 12)

ANS: (i) (c), The path is an ellipse

2a = 26

⇒ a = 13, 2c = 10 ⇒ c = 5
70

c2 = a2 – b2

⇒ 25 = 169 – b2

⇒ b2 = 144

⇒ b = 12

(ii) (c),

(iii) (a),

(iv) (b), Coordinates of vertices = (± a, 0) = (± 13, 0)

(v) (a) Coordinates of foci = (± 5, 0)

2. A man visited India gate and moving on a path, he observed that his distance from India gate is always 500 metres.
Taking India gate as origin. Answer the following:

(i) Find the equation of path of the man.

(ii) If he saw a balloon seller is at point (600, 800) then the man has to move towards India gate or away from it.

(iii) If man is considered to be at point (400, 300), then find the distance between balloon seller and man.

(iv) Find the locus of balloon seller if he also start moving around India gate on circular path from the point where
he was standing.

(v) Find the area of triangle formed by joining the India gate, the position of man and the position of balloon
seller.
2 2
ANS: (i) x + y = 250000 (ii) Man has to move away from India gate (iii)
2 2
(iv)x + y = 1000000 (v) Area = 70000 m2
3.
A beam is supported at its ends by supports which are 12 m apart. Since the load is concentrated at its centre, there is a
deflection of 3 cm at the centre and defected beam is in the shape of parabola. Now considering the centre of beam is at
origin as shown in figure. Answer the following:
71

(i) The equation of parabola will be of the form

(a) y2 = 4ax (b) x2 = 4ay

(c) y2 = – 4ax (d) x2 = – 4ay

(ii) The focus of parabola is

(a) (0, 300) (b) (0, 3)

(c) (3, 0) (d) (12, 0)

(iii) The length of latus rectum of parabola is

(a) 1200 m (b) 1200 cm

(c) 12 m (d) 48 m

(iv) The points on the parabola where it is supported by two supports

(a) (± 6, 3) (b)

(c) (d) (d) (3, 6)

(v) How far from the centre is the deflection 1 cm?

(a) (b)

(c) (d)
2
ANS: (i) (b), Equation of parabola is x = 4ay

(ii) (a), x2 = 4ay

(iii) (a), Length of latus rectum is = 4a

= 4 × 300

= 1200 m

(iv) (c),

(v) (a), Where the deflection is 1 cm. Let the coordinates of point be
72

1.A beam is supported at its ends by supports which are 12 metres apart. Since the load is concentrated at its centre,
there
is a deflection of 3 cm at the centre and the deflected beam is in the shape of a parabola. How far from the centre is the
deflection 1 cm?
Solution Let the vertex be at the lowest point and the axis vertical. Let the coordinate
axis be chosen as

The equation of the parabola takes the form x2 = 4ay.


Since it passes through (6, 3/100)
2
we have (6) = 4a(3/100) i.e. a=300 m
let AB is the deflection of the beam which is 1/100 m
So,. Coordinates of B are (x,2/100)
Therefore x2 = 4 × 300 × 2/100=24
i.e. x =√ 24 metres

2. If the latus rectum of an ellipse with axis along x-axis and centre at origin is 10, distance between foci = length of
minor axis, then the equation of the ellipse is
Ans Given that 2b2/a =10
= and 2ae = 2b b = ae and
Again, we know that
b2 = a2 (1 – e2)
or 2 a2 e2 = a2 √
Thus a = b √2
Again 2 b2/a= 10
Or b=5√





73

An equilateral triangle is inscribed in the parabola y2 = 4ax whose one vertex is at the vertex of the parabola. Find the
length of the side of the triangle.
Solution As shown in the figure APQ denotes the equilateral
triangle with its equal
sides of length l (say).
Here AP = l so AR = l cos30°=l√3/2
.
Also, PR = l sin 30° = l/2
.
Thus (√3/2,1/2 )are the coordinates of the point P lying on the
parabola y2 = 4ax.
Therefore,
l2/4=4a(l√3/2)l=8a√3

Thus, 8 a√3 is the required length of the side of the equilateral


triangle inscribed in the parabola y2 = 4ax.
74

(Chapter -11 : INTRODUCTION TO THREE DIMENSIONAL


GEOMETRY)
MCQ QUESTIONS
Q.1: Which octant do the point (−5,4,3) lie?

A. Octant I

B. Octant II

C. Octant III

D. Octant IV

Q.2: A point is on the x-axis. Which of the following represent the point?

A. (0, x, 0)

B. (0, 0, x)

C. (x, 0, 0)

D. None of the above

Q.3: Coordinate planes divide the space into ______ octants.

A. 4

B. 6

C. 8

D. 10

Q.4: What is the distance between the points (2, –1, 3) and (–2, 1, 3)?

A. 2√5 units

B. 25 units

C. 4√5 units

D. √5 units

Q.5: The maximum distance between points (3sin θ, 0, 0) and (4cos θ, 0, 0) is:

(a) 3 units
75

(b) 4 units

(c) 5 units

(d) Cannot be determined

Assertion and Reason Questions


Directions: Each of these questions contains two statements, Assertion and Reason. Each of these
questions also has four alternative choices, only one of which is the correct answer. You have to select one
of the codes (a) (b), (c) and (d) given below.

(a) Assertion is correct, reason is correct, reason is a correct explanation for assertion

(b) Assertion is correct, reason is correct, reason is not a correct explanation for assertion

(c) Assertion is correct, reason is incorrect

(d) Assertion is incorrect, reason in correct

1 Assertion: The coordinates of the point which divides the join of A(2,-1,4) and B(4,3,2) in the ratio 2:3
externally is C(-2,-9,8)

Reason: If P(x₁,y₁,z₁) and Q(x₂,y₂.z₂) be two points, and let R be a point on PQ produced dividing it externally
in the ratio m₁: m₂. Then the coordinates of R are

( 𝑚1 𝑥2 −𝑚2 𝑥1 𝑚1 𝑦2 −𝑚2 𝑦1 𝑚1 𝑧2 −𝑚2 𝑧1


𝑚1 −𝑚2
, 𝑚1 −𝑚2
, 𝑚1 −𝑚2
)
2. Assertion: If three vertices of a parallelogram ABCD are A(3,-1,2), 8(1,2,-4) and C(-1.1.2). then the fourth
vertex is (1.-2,8).

Reason: Diagonals of a parallelogram bisect each other and mid-point of AC and BD coincide

TWO MARKS QUESTIONS


1. Find the distance between the points P(-2,4,1) and Q(1, 2, – 5).
2. Write the distance of point P(2,3,5) from the xy-plane.
3. Name the octants in which the following points lie: (-5,-4,7), (-7,2,-5)
4. How far apart are the points (2, 0, 0) and (-3, 0, 0)
5. Write the distance of point P(5,12,13) from the x-axis.
THREE MARKS QUESTIONS
1. Find the equation of the set of points which are equidistant from the points (1, 2, 3) and (3, 2, –1).
76

2. Find the coordinates of a point equidistant from the four points O (0,0,0), A(a, 0, 0), B(0, b, 0) and C(0,0,
c).

3. Prove that the points (a,b,c), (b,c,a) and (c,a,b) are the vertices of an equilateral triangle.

4. Show that the points (-2, 3, 5), (1, 2, 3) and (7, 0, -1) are collinear.

5. What are the coordinates of the vertices of a cube whose edge is 2 units, one of whose vertices coincides
with the origin and the three edges passing through the origin, coincide with the positive direction of the axes
through the origin?

FIVE MARKS QUESTIONS


1. Three consecutive vertices of a parallelogram ABCD are .4(6, -2,4), 6(2,4, -8), C(-2, 2, 4). Find the
coordinates of the fourth vertex.

2. Show that the triangle ABC with vertices .4(0,4,1), 6(2,3, -1) and C(4, 5,0) is right angled.

3. The mid-point of the sides of a triangle are (1, 5, -1), (0,4, -2) and (2, 3,4). Find its vertices. Also, find the
centroid of the triangle.

CASE BASED QUESTIONS

1. A boy is standing at point O and observe three kites A, B and C in space. Taking O as origin if the
coordinates of three kites A, B and C are (3, 4, 5), (–1, 3, 4) and (2, – 1, 4) respectively, then
(i) the distance between kites A and B will be —-
(ii) The coordinates of a point on the y-axis which is at a distance of units from kite A are
(a) (0, 0, 5) (b) (0, 7, 0)
(c) (3, 0, 0). (d) (0, 5, 0)
(iii) The coordinates of point D so that ABCD is a parallelogram are
(a) (6, 0, 5) (b) (4, 9, 5)
(c) (–1, 3, 2) (d) (5, 6, 0)
(iv) If the points (0, –1, –7), (2, 1, –9) and (6, 5, –13) represent kites A, B and C then the kites
(a) are collinear
(b) form right angled triangle
(c) form an isosceles triangles
(d) form a rhombus

3.
77

(Chapter -12 : LIMITS AND DERIVATIVES)


MCQs
QN SOLUTION MARKS
1 𝟒𝑿𝟐 −𝟏 1
The value of 𝐥𝐢𝐦𝑿→𝟏 ( 𝟐𝑿−𝟏 ) =
𝟐
(A) 1 (B) 2 (C) 3 (D) 4
2 𝒕𝒂𝒏𝟐 𝟑𝒙 1
The value of 𝐥𝐢𝐦𝑿→𝟎 ( 𝑿𝟐 ) =
(A) 0 (B) 3 (C) ∞ (D) 9
3 𝑰𝒙−𝟗𝑰 1
If f(x) = , then find 𝐥𝐢𝐦𝒙→33 𝒇(𝒙)
𝒙−𝟗
(A) 1 (B) -1 (C) 2 (D) 0
4 𝟑 𝟐 1
The value of 𝐥𝐢𝐦𝐱→𝟑 (𝟒𝒙 − 𝟐𝒙 − 𝒙 + 𝟏) =
(A) 40 (B) 20 (C) 38 (D) 88
5 If f(x) = 𝒙𝟐 − 𝟓𝒙 + 𝟕, then f' (3) is : 1
(A) 11 (B) 1 (C) 4 (D) 0

ASSERTION - REASON QUESTIONS


Q N QUESTION MARKS
Directions (Q Nos. - 6&7)
In the questions given below are two statements labelled as Assertion (A) and Reason
(R). In the context of the two statements, which one of the following is correct ?
(a) Both A and B are correct, R is the correct explanation of A
(a) Both A and B are correct, R is not the correct explanation of A
(c) A is correct, R is incorrect
(d) A is incorrect, R is correct
𝑿𝟐 − 𝟏 𝟑 1
6 Assertion : The value of 𝐥𝐢𝐦𝑿→𝟐+ (𝟐𝑿 + 𝟒) = 𝟖

Reason : 𝐥𝐢𝐦𝑿→𝒂+ 𝒇(𝒙) = 𝐥𝐢𝐦𝒉→𝟎 𝒇(𝒂 + 𝒉)

7 Assertion : The derivative of f(x) = cos x at x = 0 is 0 1

Reason : 𝐅𝐨𝐫 𝐚 𝐟𝐮𝐧𝐜𝐭𝐢𝐨𝐧 𝒇(𝒙),


f(a+h)− f(a)
f'(a) = 𝐥𝐢𝐦𝒉→𝟎 h

VSA
8 𝑰𝒙𝑰 2
Evaluate LHL and RHL of the𝐥𝐢𝐦 𝒙
𝐱→𝟎

9 𝑿𝟐𝟏 − 𝟏 2
Evaluate : 𝐥𝐢𝐦
𝐱→𝟏 𝑿𝟑 − 𝟏

𝒕𝒂𝒏 𝒙
10 Evaluate : 𝐥𝐢𝐦 2
𝐱→𝟎 𝒙

11 Find the derivative of x2 - 3 at x = 5 2

12 Compute the derivative of f(x) = sin2x 2


78

Q N SA QUESTION MARKS
13 𝒕𝒂𝒏 𝒙 −𝟐 𝒕𝒂𝒏 𝟑𝒙 + 𝒕𝒂𝒏 𝟓𝒙 3
Evaluate : 𝐥𝐢𝐦 [ ]
𝐱→𝟎 𝒙

14 𝒙𝟒 − 𝟖𝟏 3
Evaluate : 𝐥𝐢𝐦 𝟐𝒙𝟐 −𝟓𝒙− 𝟑
𝐱→𝟑

15 Differentiate 2x3 - 4x2 + 6x + 8 w.r.t. x 3

16 𝟏 3
Differentiate(𝒙 + )𝟑 w.r.t. x
𝒙

17 Compute the derivative of sin x. 3

QN LA QUESTION MARKS
18 𝒙𝟑 −𝟕𝒙 𝟐 + 𝟏𝟓𝒙−𝟗 5
Evaluate :𝐥𝐢𝐦 𝒙𝟒 −𝟓𝒙 𝟑 + 𝟐𝟕𝒙−𝟐𝟕
𝐱→𝟑

19 𝒎𝒙 𝟐 + 𝒏, 𝒙 < 0 5
If f(x) = {𝒏𝒙 + 𝒎, 𝟎 ≤ 𝒙 ≤ 𝟏,
𝒏𝒙 𝟑 + 𝒎, 𝒙 > 1
𝒇𝒐𝒓 𝒘𝒉𝒂𝒕 𝒊𝒏𝒕𝒆𝒈𝒆𝒓𝒔 𝒎 𝒂𝒏𝒅 𝒏 𝒅𝒐𝒆𝒔 𝒃𝒐𝒕𝒉 𝐥𝐢𝐦 𝐟(𝐱) and 𝐥𝐢𝐦 𝐟(𝐱) exist ?
𝐱→𝟎 𝐱→𝟏
20 𝟒𝒙 + 𝟓 𝒔𝒊𝒏 𝒙 5
Find the derivative of : f(x) =
𝟑𝒙 + 𝟕 𝒄𝒐𝒔 𝒙

Q N CASE STUDY QUESTION MARKS


21 4

Mr. Shami, a maths teacher of class XI wants to check the knowledge of his students,
so he writes a function f(x) = sin x + cos x, where f be a real valued function defined
by
𝐟(𝐱 +𝐡)− 𝐟(𝐱 )
f'(x) = = 𝐥𝐢𝐦𝒉→𝟎 𝐡
For a function f(x) = sin x + cos x, answer the following questions ;
𝒅
(a) Find 𝒅𝒙 {f'(x)} at x = 90°
(b) If f(x) = cos2 x - sin2 x, then find the value of f'(45°)
22 𝒈(𝒙)
A function f is said to be a rational function, if f(x) = , where g(x) and h(x) are
𝒉(𝒙)
polynomial functions such that h (x) ≠ 0.
𝒈(𝒙) 𝒍𝒊𝒎 𝒈(𝒙) 𝒈(𝒂)
𝒙→𝒂
Then 𝐥𝐢𝐦 f(x) = 𝐥𝐢𝐦 = =
𝐱→𝐚 𝐱→𝐚 𝒉(𝒙) 𝒍𝒊𝒎 𝒉(𝒙) 𝒉(𝒂)
𝒙→𝒂
79

However, if 𝒉(𝒂) = 0, then there are two cases arise :


(i) g(a) ≠ 0 (ii) 𝒈(𝒂) = 0
In the first case, we say that the limit does not exist.
In the second case, we can find limit.
Based on the above information, answer the following questions :
(a) Find the value of 2
𝒙𝟐−𝟒
𝐥𝐢𝐦 𝟑
𝐱→𝟐 𝒙 − 𝟒𝒙 𝟐 + 𝟒𝒙
2
(b) Find the value of
𝒙 𝟑 − 𝟒𝒙 𝟐 + 𝟒𝒙
𝐥𝐢𝐦
𝐱→𝟐 𝒙𝟐−𝟒
23 4

One day Virat, a friend of Rohit, explain him a direct application mehod to find the
derivative of polynomials. He explained that :
Let f(x) = an xn+ an-1 xn-1 + … + a1 x + a0 be a polynomial function, where a1s are all
real numbers and aa ≠ 0. Then, the derivative function is given by
𝒅𝒇(𝒙)
= 𝒏an xn - 1 + (n - 1) an -1 xn - 2 + … + 2a2 x + a1.
𝒅𝒙
After explaining the method, he ask these two questions to Rohit :
(a) Compute the derivative of : 5x100 - x48 + 1.
(b) Find the derivative of : f(x) = 1 + x + x2 + x3 + … + x50 at x = 1.
Help Rohit to solve these questions.

QN MCQs SOLUTIONS MARKS


1 (B) 2 1
2 (D) 9 1
3 (B) -1 1
4 (D) 88 1
5 (B) 1 1
SOLUTION (ASSERTION - REASON QUESTIONS)
(a) Both A and B are correct, R is the correct explanation of A 1
6
7 (a) Both A and B are correct, R is the correct explanation of A 1
80

SOLUTION (VSA)
8 𝑰𝒙𝑰 −𝒙 1
LHL = 𝐥𝐢𝐦− 𝒙 = 𝐥𝐢𝐦 𝒙 = -1
𝐱→𝟎 𝐱→𝟎
𝑰𝒙𝑰 𝒙
RHL = 𝐥𝐢𝐦+ = 𝐥𝐢𝐦 𝒙 = 1 1
𝐱→𝟎 𝒙 𝐱→𝟎

9 𝑿𝟐𝟏 − 𝟏 𝑿𝟐𝟏 − 𝟏 𝑿𝟑 − 𝟏 𝑿𝟐𝟏 − 𝟏 𝑿𝟑 − 𝟏 1/2


𝐥𝐢𝐦 = 𝒍𝒊𝒎 [ ÷ ] = 𝒍𝒊𝒎 [ ] ÷ 𝒍𝒊𝒎 [ 𝑿− 𝟏 ]
𝐱→𝟏 𝑿𝟑 − 𝟏 𝒙→𝟏 𝑿− 𝟏 𝑿− 𝟏 𝒙→𝟏 𝑿− 𝟏 𝒙→𝟏
1/2
20 2 𝑿𝒏 −𝒂𝒏 n–1
= 21 (1) ÷ 3 (1) using 𝐥𝐢𝐦 = na
𝐱→𝐚 𝑿− 𝒂 1/2
= 21 ÷ 3 =7 1/2

10 𝒕𝒂𝒏 𝒙 𝒔𝒊𝒏 𝒙 1/2


We have 𝐥𝐢𝐦 = 𝐥𝐢𝐦 𝒙 𝒄𝒐𝒔 𝒙
𝐱→𝟎 𝒙 𝐱→𝟎
𝒔𝒊𝒏 𝒙 𝟏 𝒔𝒊𝒏 𝒙
= 𝐥𝐢𝐦 . 𝐥𝐢𝐦 using 𝐥𝐢𝐦 = 1, using 𝐥𝐢𝐦cos x = 1 1
𝐱→𝟎 𝒙 𝐱→𝟎 𝒄𝒐𝒔 𝒙 𝐱→𝟎 𝒙 𝐱→𝟎

= 1.1 = 1 1/2

11 Let f(x) = x2 - 3, Accordingly,


𝐟(𝟓+𝐡)− 𝐟(𝟓) 1/2
F' (5) =𝐥𝐢𝐦𝒉→𝟎 𝐡
[(𝟓+𝒉)𝟐 − 𝟑]−[(𝟓)𝟐 − 𝟑]
= 𝐥𝐢𝐦𝒉→𝟎 𝐡
(𝟓)𝟐 + 𝟐 . 𝟓 . 𝒉+ (𝒉)𝟐 − 𝟑 − (𝟓)𝟐 + 𝟑
= 𝐥𝐢𝐦𝒉→𝟎 𝐡 𝟏
𝟏𝟎𝒉 + (𝒉)𝟐
= 𝐥𝐢𝐦𝒉→𝟎 𝐡

= 𝐥𝐢𝐦 ( 10 + h) = 10
𝒉→𝟎 1/2

12 𝒅𝒇(𝒙) 𝒅(𝒔𝒊𝒏 𝒙.𝒔𝒊𝒏 𝒙) 1/2


= useing Leibnitz product rule
𝒅𝒙 𝒅𝒙 1/2
= (sin x)' sin x + sin x (sin x)'
1/2
= (cos x) sin x + sin x (cos x)
1/2
= 2 sin x cos x = sin 2x

Q N SA SOLUTIONS MARKS
13 𝒕𝒂𝒏 𝒙 − 𝟐 𝒕𝒂𝒏 𝟑𝒙 + 𝒕𝒂𝒏 𝟓𝒙
𝐥𝐢𝐦 [ ]
𝐱→𝟎 𝒙
𝒕𝒂𝒏 𝒙 𝟐𝒕𝒂𝒏 𝟑𝒙 𝒕𝒂𝒏 𝟓𝒙
= 𝐥𝐢𝐦 [ − + ]
𝐱→𝟎 𝒙 𝒙 𝒙 1
𝒕𝒂𝒏 𝒙 𝒕𝒂𝒏 𝟑𝒙 𝒕𝒂𝒏 𝟓𝒙
= 𝐥𝐢𝐦 [ −𝟔 + 𝟓 ]
𝐱→𝟎 𝒙 𝟑𝒙 𝟓𝒙
𝒕𝒂𝒏 𝒙 𝒕𝒂𝒏 𝟑𝒙 𝒕𝒂𝒏 𝟓𝒙
= 𝐥𝐢𝐦 [ ] - 6 𝐥𝐢𝐦 [ ] + 5 𝐥𝐢𝐦 [ ]
𝐱→𝟎 𝒙 𝐱→𝟎 𝟑𝒙 𝐱→𝟎 𝟓𝒙
𝒕𝒂𝒏 𝒙 𝒕𝒂𝒏 𝟑𝒙 𝒕𝒂𝒏 𝟓𝒙
= 𝐥𝐢𝐦 [ ] -6 𝐥𝐢𝐦 [ ] + 5 𝐥𝐢𝐦 [ ]
𝐱→𝟎 𝒙 𝟑𝐱→𝟎 𝟑𝒙 𝟓𝐱→𝟎 𝟓𝒙
1
= 1 - 6(1) + 5(1) = 0
1
14 Putting x = 3
𝒙𝟒 − 𝟖𝟏 𝟑𝟒 − 𝟖𝟏 𝟖𝟏 − 𝟖𝟏 𝟎 𝟎
𝐥𝐢𝐦 𝟐𝒙𝟐 −𝟓𝒙− 𝟑 = = 𝟏𝟖−𝟏𝟓− 𝟑= 𝟎 since, it is of the form 𝟎
𝐱→𝟑 𝟐(𝟑𝟐 )−𝟓(𝟑)− 𝟑
81

We simplify as 1
𝒙𝟒 − 𝟖𝟏 (𝒙𝟐 )𝟐 − 𝟗𝟐 (𝒙−𝟑)(𝒙+𝟑)(𝒙𝟐 + 𝟗) (𝒙+𝟑)(𝒙𝟐 + 𝟗)
𝐥𝐢𝐦 𝟐𝒙𝟐 −𝟓𝒙− 𝟑 = 𝐥𝐢𝐦 𝟐𝒙𝟐 −𝟔𝒙 + 𝒙 − 𝟑 = 𝐥𝐢𝐦 (𝒙−𝟑)(𝟐𝒙+𝟏)
= 𝐥𝐢𝐦 (𝟐𝒙+𝟏)
𝐱→𝟑 𝐱→𝟑 𝐱→𝟑 𝐱→𝟑 1
Putting x = 3
(𝟑+𝟑)(𝟑𝟐 + 𝟗) 𝟔 (𝟗+𝟗) 𝟔 (𝟏𝟖) 𝟏𝟎𝟖
= = = =
𝟐(𝟑)+ 𝟏 𝟔+𝟏 𝟕 𝟕
1
15 On differentiating both sides w.r.t. x. We get
𝒅𝒚 𝒅(𝟐𝒙𝟑 − 𝟒𝒙𝟐 + 𝟔𝒙 + 𝟖 ) 𝒅 𝒅 𝒅 𝒅 𝟏
= =2 𝒅𝒙x3 - 4 𝒅𝒙x2 + 6𝒅𝒙 (x) + 𝒅𝒙(8) 1𝟐
𝒅𝒙 𝒅𝒙

= 2(3x2) - 4(2x) + 6(1) + 0 = 6x2 - 8x + 6 1𝟐


𝟏

16 𝟏 𝟏 𝟏 𝟏 𝟑 𝟏 1
Let y = (𝒙 + 𝒙 )𝟑 = x3+ 3x (𝒙𝟐 ) +𝟑𝒙𝟐 (𝒙 )+ 𝒙𝟑 = x3 + 𝒙 + 3x + 𝒙𝟑
On differentiating both sides w.r.t. x. We get
𝒅𝒚 𝒅 𝒅 𝟑 𝒅 𝒅 𝟏
= 𝒅𝒙 x3 + 𝒅𝒙(𝒙) + 𝒅𝒙 (3x) + 𝒅𝒙(𝒙𝟑 )
𝒅𝒙
𝒅 𝒅 𝒅 𝟏
= 𝟑𝒙𝟐 +3 𝒅𝒙x-1 + 3 𝒅𝒙 x + 𝒅𝒙x-3 1𝟐
𝟑 𝟑
=
𝟑𝒙𝟐 - 𝒙𝟐 + 3 - 𝒙𝟒
1/2

17 Let f(x) = sin x, then


𝒅𝒇(𝒙) 𝐟(𝐱 +𝐡)− 𝐟(𝐱 ) 𝐬𝐢𝐧(𝐱+𝐡)− 𝐬𝐢𝐧(𝐱) 1
= 𝐥𝐢𝐦𝒉→𝟎 = 𝐥𝐢𝐦𝒉→𝟎
𝒅𝒙 𝐡 𝐡
𝟐𝒙+𝒉 𝒉
𝟐 𝐜𝐨𝐬 ( )− 𝐬𝐢𝐧( )
= 𝐥𝐢𝐦𝒉→𝟎 𝟐 𝟐
using formula for sin A - sin B 1
𝐡
𝒉
𝒉 𝒔𝒊𝒏
= 𝐥𝐢𝐦 𝐜𝐨𝐬 ( x + 𝟐 ) . 𝐥𝐢𝐦 𝒉
𝟐
= cos x . 1 = cos x 1
𝒉→𝟎 𝒉→𝟎
𝟐

QN LA SOLUTIONS MARKS
18 𝒙𝟑 −𝟕𝒙 𝟐 + 𝟏𝟓𝒙−𝟗 𝟎 1
When x = 3, the expression 𝐥𝐢𝐦 𝒙𝟒 −𝟓𝒙 𝟑 + 𝟐𝟕𝒙−𝟐𝟕 assumes the form 𝟎
𝐱→𝟑
So, (x - 3) is a factor of numerator and denominator
Now, factorizing the numerator and denominator we get
𝒙𝟑 − 𝟕𝒙 𝟐 + 𝟏𝟓𝒙 − 𝟗
𝐥𝐢𝐦 𝟒
𝐱→𝟑 𝒙 − 𝟓𝒙 𝟑 + 𝟐𝟕𝒙 − 𝟐𝟕 2
( 𝒙−𝟑) ( 𝒙 𝟐 − 𝟒𝒙+ 𝟑)
= 𝐥𝐢𝐦 ( 𝒙−𝟑) (𝒙𝟑 −𝟐𝒙 𝟐 − 𝟔𝒙 +𝟗)
𝐱→𝟑
( 𝒙 𝟐 − 𝟒𝒙+ 𝟑) 𝟎
= 𝐥𝐢𝐦 (𝒙𝟑 −𝟐𝒙 𝟐 − 𝟔𝒙 +𝟗)form𝟎
𝐱→𝟑
( 𝒙−𝟑) ( 𝒙−𝟏)
= 𝐥𝐢𝐦 ( 𝒙−𝟑) (𝒙 𝟐 + 𝒙 −𝟑) 2
𝐱→𝟑
( 𝒙−𝟏) 𝟑−𝟏 𝟐
= 𝐥𝐢𝐦 = =𝟗
𝐱→𝟑 (𝒙 𝟐 + 𝒙 −𝟑) 𝟗+𝟑−𝟑
19 𝒎𝒙 𝟐 + 𝒏, 𝒙 < 0
The given function is f(x) = {𝒏𝒙 + 𝒎, 𝟎 ≤ 𝒙 ≤ 𝟏,
𝒏𝒙 𝟑 + 𝒎, 𝒙 > 1
𝐥𝐢𝐦− f(x) = 𝐥𝐢𝐦 (𝒎𝒙 𝟐 + 𝒏) = m(0)2 + n = n 1
𝐱→𝟎 𝐱→𝟎
𝐥𝐢𝐦 f(x) = 𝐥𝐢𝐦(𝒏𝒙 + 𝒎) = n(0) + m = m
𝐱→+ 𝐱→𝟎
82

Thus 𝐥𝐢𝐦 f(x) exists if m = n 𝟏


𝐱→𝟎 1𝟐

𝐥𝐢𝐦 f(x) = 𝐥𝐢𝐦 (𝒏𝒙 + 𝒎) = n(1) + m = m + n


𝐱→𝟏− 𝐱→𝟏
𝐥𝐢𝐦+ f(x) = 𝐥𝐢𝐦 (𝒏𝒙 𝟑 + 𝒎) = n (1)3 + m = m + n 1
𝐱→𝟏 𝐱→𝟏
So, 𝐥𝐢𝐦− f(x) = 𝐥𝐢𝐦+ f(x) = 𝐥𝐢𝐦 f(x) .
𝐱→𝟏 𝐱→𝟏 𝐱→𝟏
𝟏
1𝟐
Thus 𝐥𝐢𝐦 f(x) exists for any integral value of m and n.
𝐱→𝟏

20 𝟒𝒙 + 𝟓 𝒔𝒊𝒏 𝒙
Let f(x) = 𝟑𝒙 + 𝟕 𝒄𝒐𝒔 𝒙 ,
By quotient rule,
𝒅 𝒅
(𝟑𝒙 + 𝟕 𝒄𝒐𝒔 𝒙) (𝟒𝒙 + 𝟓 𝒔𝒊𝒏 𝒙)− (𝟒𝒙 + 𝟓 𝒔𝒊𝒏 𝒙) (𝟑𝒙 + 𝟕 𝒄𝒐𝒔 𝒙)
f’(x) = 𝒅𝒙 𝒅𝒙 2
(𝟑𝒙 + 𝟕 𝒄𝒐𝒔 𝒙)2
𝒅 𝒅 𝒅 𝒅
(𝟑𝒙 + 𝟕 𝒄𝒐𝒔 𝒙)[𝟒 (𝒙)+𝟓 (𝒔𝒊𝒏 𝒙)] − (𝟒𝒙 + 𝟓 𝒔𝒊𝒏 𝒙)[𝟑 (𝒙)+𝟕 (𝒄𝒐𝒔 𝒙)]
𝒅𝒙 𝒅𝒙 𝒅𝒙 𝒅𝒙
= (𝟑𝒙 + 𝟕 𝒄𝒐𝒔 𝒙)2
(𝟑𝒙 + 𝟕 𝒄𝒐𝒔 𝒙)( 𝟒+𝟓 𝒄𝒐𝒔 𝒙) − (𝟒𝒙 + 𝟓 𝒔𝒊𝒏 𝒙)(𝟑−𝟕 𝒔𝒊𝒏 𝒙)
= 2
(𝟑𝒙 + 𝟕 𝒄𝒐𝒔 𝒙)2
𝟏𝟐𝒙 + 𝟏𝟓𝒙 𝒄𝒐𝒔 𝒙 + 𝟐𝟖 𝒄𝒐𝒔 𝒙 + 𝟑𝟓𝒄𝒐𝒔 𝟐 𝒙−𝟏𝟐𝒙 + 𝟐𝟖𝒙 𝒔𝒊𝒏 𝒙−𝟏𝟓 𝒔𝒊𝒏 𝒙 + 𝟑𝟓 𝒔𝒊𝒏 𝟐 𝒙
= (𝟑𝒙 + 𝟕 𝒄𝒐𝒔 𝒙)2
𝟏𝟓𝒙 𝒄𝒐𝒔 𝒙 + 𝟐𝟖 𝒄𝒐𝒔 𝒙 + 𝟐𝟖𝒙 𝒔𝒊𝒏 𝒙−𝟏𝟓 𝒔𝒊𝒏 𝒙 + 𝟑𝟓
= (𝟑𝒙 + 𝟕 𝒄𝒐𝒔 𝒙)2
𝟏𝟓𝒙 𝒄𝒐𝒔 𝒙 + 𝟐𝟖 𝒄𝒐𝒔 𝒙 + 𝟐𝟖𝒙 𝒔𝒊𝒏 𝒙−𝟏𝟓 𝒔𝒊𝒏 𝒙 + 𝟑𝟓( 𝒔𝒊𝒏 𝟐 𝒙 + 𝒄𝒐𝒔 𝟐 𝒙)
= 1
(𝟑𝒙 + 𝟕 𝒄𝒐𝒔 𝒙)2
𝟏𝟓𝒙 𝒄𝒐𝒔 𝒙 + 𝟐𝟖 𝒄𝒐𝒔 𝒙 + 𝟐𝟖𝒙 𝒔𝒊𝒏 𝒙−𝟏𝟓 𝒔𝒊𝒏 𝒙 + 𝟑𝟓
= (𝟑𝒙 + 𝟕 𝒄𝒐𝒔 𝒙)𝟐

Q N CASE STUDY QUESTIONS MARKS


21 (a) We have, f'(x) = cos x - sin x 2
𝒅 𝒅
So, 𝒅𝒙 {f'(x)} = 𝒅𝒙 (cos x - sin x)
𝒅 𝒅
= 𝒅𝒙 ( cos x)𝒅𝒙 - ( sin x)
= - sin x - cos x
𝒅
So, 𝒅𝒙 {f'(x)} at x = 90°
= - sin 90° - cos 90°
=-1-0=-1
(b) We have f(x) = cos2 x - sin2 x
2
Then, f'(x) = - 2 cos x.sin x - 2 sin x.cos x
= - 4 cos x.sin x
𝟏 𝟏 −𝟒
So, f'(45°) = -4 x x = 𝟐 = -2
√𝟐 √𝟐
22 𝒙 𝟐 −𝟒 2
(a) Consider f(x) = 𝒙 𝟑 −𝟒𝒙 𝟐+ 𝟒𝒙
On putting x = 2, we get
𝟐 𝟐 −𝟒 𝟒−𝟒 𝟎 𝟎
f(2) = = 𝟖−𝟏𝟔+𝟖 = 𝟎 i.e., it is of the form
𝟐 𝟑 − 𝟒(𝟐 𝟐 )+ 𝟒(𝟐) 𝟎
so, let us first factories it
𝒙 𝟐 −𝟒 (𝒙+𝟐)(𝒙−𝟐) (𝒙+𝟐) 𝟐+ 𝟐 𝟒
consider, 𝐥𝐢𝐦 = 𝐥𝐢𝐦 = 𝐥𝐢𝐦 = 𝟐 (𝟐−𝟐) = 𝟎
𝐱→𝟐 𝒙 𝟑 −𝟒𝒙 𝟐 + 𝟒𝒙 𝐱→𝟐 𝒙 (𝒙−𝟐) 𝟐 𝐱→𝟐 𝒙 ( 𝒙−𝟐)
which is not defined.
𝒙 𝟐 −𝟒
So, 𝐥𝐢𝐦 does not exist. 2
𝐱→𝟐 𝒙 𝟑 −𝟒𝒙 𝟐 + 𝟒𝒙
𝟎
(b) Evaluating the function at 2, it is of the form 𝟎 .
83

𝒙 𝟑 − 𝟒𝒙 𝟐 + 𝟒𝒙 𝒙 (𝒙−𝟐) 𝟐 𝒙(𝒙−𝟐) 𝟐( 𝟐 − 𝟐) 𝟎
Hence 𝐥𝐢𝐦 = 𝐥𝐢𝐦 = 𝐥𝐢𝐦 = = =0
𝐱→𝟐 𝒙 𝟐 −𝟒 𝐱→𝟐 (𝒙+𝟐)(𝒙−𝟐) 𝐱→𝟐 ( 𝒙 +𝟐) (𝟐 +𝟐) 𝟒

23 (a) Consider the polynomial : 5x100 - x48 + 1 2


A direct application of the above method gives the derivative of given polynomial
500 x99- 48 x47
(b) Consider the function : f(x) = 1 + x + x2 + x3 + … + x50 at x = 1
A direct application of the above method gives the derivative of given function
= 1 + 2x +3 x2 + 4x3 + … + 50x49 2
At x = 1 the value of this function equals :
(𝟓𝟎)(𝟓𝟏)
1 + 2 + 3 + … + 50 = 𝟐 = 1275

3.3
84

(Chapter -13 : STATISTICS)


CONCEPTS AND RESULTS

* * Mean x   i
x
n

 i i
fx
 fi
 a    i i  h, where a is the assumed mean , h is the class size and u i  i
 fu  x a
 f  h
 i 
th
 n 1
* * Median    observatio ns arranged in ascending or descending order  & the number of observatio ns
 2 
is odd.
th th
n n 
 mean of   &   1 observatio ns arranged in ascending or descending order  & the number
2 2 
of observatio ns is odd.
n 
  cf 
 l 2   h where , l  lower limit of median class, n  number of observatio ns,
 f 
 
 
cf  cumulative frequency of class preceding the median class, f  frequency of median class,
h  class size .
 f1  f 0 
* * Mode  l     h , where l  lower limit of the modal class, h  size of the class interval,
 2f1  f 0  f 2 
f1  frequency of the modal class, f 0  frequency of the class preceding the modal class,
f 2  frequency of the class succeeding the modal class.
** Measures of Dispersion: The dispersion or scatter in a data is measured on the basis of the observations
and the types of the measure of central tendency, used there. There are following measures of dispersion:
(i) Range, (ii) Quartile deviation, (iii) Mean deviation, (iv) Standard deviation.
** Range: Range of a series = Maximum value – Minimum value.
** Mean Deviation : The mean deviation about a central value ‘a’ is the mean of the absolute values of the
deviations of the observations from ‘a’. The mean deviation from ‘a’ is denoted as M.D. (a).
(i) For ungrouped data
1 n 1 n
M.D.( x )   x i  x , where x  Mean
n i 1
M.D.(M)   x i  x , where M  Median
n i 1
(ii ) For grouped data
(a) Discrete frequency distributi on
n
 fi x i  x 1 n 1 n
M.D.( x )  i 1
n
  fi x i  x
N i 1
M.D.(M)   xi  M
N i 1
 fi
i 1
85

(b) Continuous frequency distributi on

fi x i  x , u sin g x  a    i i  h
1 n  fu 
M.D.( x )  
N i 1  f 
 i 

n 
1 n   cf 
M.D.(M)   x i  M , u sin g M  l   2 h
N i 1  f 
 

Section A (MCQ)
Q.1 If the variance of the data is 121, the standard deviation of the data is:

(a) 121 (b) 11 (c) 12 (d) 21

Answer: (b) 11

Q.2Find the median of 36, 72, 46, 42, 60, 45, 53, 46, 51, 49.

A. 42 B. 45.5 C. 47.5 D. 45

Answer: C. 47.5

Q.3 Range of a data is equal to:

(a) Range = Max Value – Min Value (b) Range = Max Value + Min Value

(c) Range = (Max Value – Min Value)/2 (d) Range = (Max Value + Min Value)/2

Answer: (a) Range = Max Value – Min Value

Q.4 Mean deviation from mean of the following data ; 4,7,8,9,10,12,13,17 is

(a ) 2 (b) 3 (c ) 4 ( d) 5

Answer : (b)

Q.5 If the mean of 1,3,4,5,7 is m ,the numbers 3,2,2,4,3,3,p have mean m-1 and median q then p + q is

(a) 4 (b) 5 (c) 6 (d) 7

Answer : (d) 7

Assertion Reason Questions


DIRECTION : In each of the following questions, a statement of Assertion is given followed by a
corresponding statement of Reason just below it. Of the statements, mark the correct answer as
(a) Both assertion and reason are true and reason is the correct explanation of assertion.
(b) Both assertion and reason are true but reason is not the correct explanation of assertion.
(c) Assertion is true but reason is false.
(d) Assertion is false but reason is true.
86

Q.1 Assertion (A) The mean deviation about mean for the data 4,7,8,9,10,12,13,17,3.
Reason (R ) The mean deviation about the mean for 38,70,48,40,42,55,63,46,54 is 8.5.
Answer :- ( c )
Q.2 Consider the following data

xi 5 10 15 20 25
fi 7 4 6 3 5
Then, the mean deviation about the mean is 6.32.
Reason ( R ) Consider the following data

xi 10 30 50 70 90
fi 4 24 28 16 8
Then, the mean deviation about the mean is 15 .
Answer :- ( c )
Q.3 Assertion ( A ) The mean deviation about median calculated for series, where variability is very
high, cannot be fully relied.
Reason ( R ) – The median is not a representative of central tendency for the series where degree of
variability is very high.
Answer :- ( a )
Section B Short Answer Type Question( 2 Marks Questions)
Question 1. Find the mean deviation about the mean.
𝑥𝑖 5 10 15 20 25
𝑓𝑖 7 4 6 3 5

Find the mean deviation about the median for the data in Exercises 7 and 8.
Question 2.
𝑥𝑖 5 7 9 10 12 15
𝑓𝑖 8 6 2 2 2 6
Solution :-
87

Question 3.
𝑥𝑖 15 21 27 30 35
𝑓𝑖 3 5 6 7 8
Solution :-

Find the mean deviation about the mean for the data in Exercises 9 and 10.
Question 4.
Income per day 0-100 100-200 200-300 300-400 400-500 500-600 600-700 700-800
Number of persons 4 8 9 10 7 5 4 3
Solution:-
88

Question 5.
Heights ( in cm) 95-105 105-115 115-125 125-135 135-145 145-155
Number of boys 9 13 26 30 12 10
Solution :-
89

( 3 Marks Questions )
Question 1.
From the prices of shares X and Y below, find out which is more stable in value:

Question 2
Find the mean and standard deviation using short-cut method
𝑥𝑖 60 61 62 63 64 65 66 67 68
𝑓𝑖 2 1 12 29 25 12 10 4 5
Solution :-
90

Find the mean and variance for the following frequency distributions in Exercises 13and 14.
Question 3
Classes 0-30 30-60 60-90 90-120 120-150 150-180 180-210
Frequencies 2 3 5 10 3 5 2

Solution :-

Question 4.
Classes 0-10 10-20 20-30 30-40 40-50
Frequencies 5 8 15 16 6
91

Solution:-

Question 5.
Find the mean, variance and standard deviation using short-cut method.
Heights in cms 70-75 75-80 80-85 85-90 90-95 95-100 100-105 105-110 110-115
No. of children 3 4 7 7 15 9 6 6 3

Solution :-
92

(5 marks Questions Question)

Question 1.
The diameters of circles (in mm) drawn in a design are given below:
Diameters 33-36 37-40 41-44 45-48 49-52
No. of Circles 15 17 21 22 25
Calculate the standard deviation and mean diameter of the circles.
[Hint: First make the data continuous by making the classes as 32.5 – 36.5, 36.5 – 40.5, 40.5 – 44.5, 44.5 –
48.5, 48.5 – 52.5 and then proceed.]
93

Question 2: The mean and variance of 8 observations are 9 and 9.25 respectively. If six of the
observations are 6 , 7 , 10, 12 , 12 and 13 . Find the remaining two observations.
Solution :-

Question 3: The mean and variance of 7 observations are 8 and 16 respectively. If five of the
observations are 2 , 4 , 10 , 12 , and 14 . Find the remaining two observations .

Solution :-
94

Section D (Case Study)


Q.1 You are gfiven the following data
95

Xi 2 5 6 8 10 12
fi 2 8 10 7 8 5
Based on the following data answer the following questions

I) Mean of the grouped data is (a) 7 (b) 7.5 (c) 8 (d) 8.5

II )Mean deviation about the mean is (a) 2.1 (b) 2.2 (c) 2.3 (d) 2.4

III ) The value of median is (A) 5 (b) 6 (c) 7 (d) 8

IV ) The mean deviation about mean is (a) 1.9 (2.0 (c) 2.2 (d)2.3

V ) The difference between mean and median is (a)0 .9 (b) 0.7 (c) 0.5 (d) 0.3

Answer – (I) b , (II) c , (III) c ,(IV) d, (V) c

Q.2. You are given some observations as 34, 66, 30, 38, 44, 50, 40, 60,42, 51.

Based on the these observations, answer the following questions.

(i) The mean of the given data is (a) 40.5 (b) 45.0 (c) 45.5 (d) 50.50

(ii) The mean deviation about mean is (a) 10 (b) 9.5 (c) 9.1 (d) 9

(iii) The mean of the data is (a) 41 (b) 42 (c) 43 (d) 44

(iv) The mean deviation about the median is (a) 8.0 (b) 8.3 (c) 8.5 (d) 8.7

(v) The difference between mean deviation about mean and mean deviation about median is

(a) 0.1 (b) 0.2 (c) 0.3 (d) 0.4

Answer :- (i) c, (ii) d, (iii) c , (iv) d , (v ) (c )

Q.3 Consider the data

xi 4 8 11 17 20 24 32

fi 3 5 9 5 4 3 1

Based on above information answer the following questions.

( i) Mean is calculated by using the formula


96

∑ fixi ∑ 𝑓𝑥 2
(𝑎) 𝑥̅ = (b) 𝑥̅ = ∑ fi xi ( c) 𝑥̅ = (d ) None of These
𝑁 𝑁

(ii) Variance is calculated by using the formula

1 1 1
(a) 𝜎 2 = ∑ fi(xi − ̅̅̅
𝑥)2 (b) 𝜎 2 = ∑ fi(xi + ̅̅̅
𝑥)2 ( c ) 𝜎 2 = ∑ fi(xi − ̅̅̅
𝑥) (d) None of these .
𝑁 𝑁 𝑁

(iii) Mean of the given data is

( a) 10 (b ) 12 (c ) 14 (d) 15

(iv) Variance of the given data is

(a) 40 (b) 45.8 (c ) 41.5 (d) 39.8

(iv) Standard deviation of the given data

( a) 6.77 ( b ) 5 (c ) 4.8 (d) 3.19

Answer :- (I ) a (ii) a ( iii ) c (iv) b (v) a


97

(Chapter -14 : PROBABILITY)


MCQs ( 5 Questions)
1-. If A and B are two events such that A ≠ ∅, B ≠ ∅, then
𝑃𝐴(∩𝐵)
(a). P (A/B) = P(A) P(B) (b) P(A/B) = 𝑃(𝐵)

(c). P(A/B) P(B/A) = 1 (d) P (A/B) = P(A) / P(B)


Ans: (b)
2-. -. If A and B are two events such that P(A) ≠ 0, and P(B/A) = 1, then

(a). A ⊂ B (b) B ⊂ A (c) B = ∅, (d) A = ∅,


Ans: (a)
3-. If P (A/B) > P(A), then which of the following is correct ?

(a). P(B/A) < P(B) (b) P(A∩ 𝐵) < P(A) . P(B)


(c) P(B/A) > P(B) (d) P(B/A) = P(B)
Ans: (c)
4-. If A and B are two events such that P(A) + P(B) - P( A and B) = P(A), then
(a). P(B/A) = 1 (b) P(A/B) = 1 (c) P(B/A) = 0 (d) P(A/B) = 0
Ans: (b)
5-. If A and B are two events such that P(A) > 0 and P(B) ≠ 1, the P(A' / B') equals
1−𝑃(𝐴∪𝐵)
(a). 1 - P(A/B) (b) 1 - P(A' / B ) (c) (d) P(A') / P(B')
𝑃(𝐵′ )

Ans: (c)

ASSERTION-REASON BASED QUESTIONS ( 2 Questions)


In the following questions, a statement of Assertion (A) is followed by a statement of Reason(R).
Choose the correct answer out of the following choices.
(A) If Both (A) and (R) are true and (R) is the correct explanation of (A).
(B) If Both (A) and (R) are true but (R) is not the correct explanation of (A).
(C) If (A) is true but (R) is false. (D) If (A) is false but (R) is true.

1-. ASSERTION (A): 20 persons are sitting in a row, two of these persons are selected at random. The
probability that the two selected person are not together is 0.9.
REASON (R) : If Ā denotes the negation of event A, then P(Ā) 1 – P(A)
98

Ans: (a)
2-. ASSERTION (A): the probability of drawing either a king or an ace from a pack of 52 playing card is
2/13.

REASON (R) : For any two events A and B P (A∪ 𝐵 ) = P(A) + P(B) - P( A - B)
Ans: (c)

2- MARKS QUESTIONS ( 5 questions)


1-. 10% of the bulbs produced in a factory are red colour and 2% are red colour and defective. If one bulb is
picked at random, determine the probability of its being defective if I red.
Ans : 1/5
2-. Three dice are thrown at the same time. Find the probability of getting three two’s if it is known that the
sum of numbers on the dice was a six.
Answer: 1/10
3-. A class consists of 80 students; 25 of them are girls and 55 boys; 10 of them has joined coaching and
remaining has not joined the coaching; 20 of them are healthy. What is the probability of selecting a healthy
coaching going girl ?
Answer : 5/512
4-. The probability that a teacher will give an un-announced test during any class meeting is 1/5.If a student is
absent twice, what is the probability that he will miss at least one test ?
Answer: 9/25
5-. A coin is tossed and a die is thrown. Find the probability that the outcome will be head or a number greater
than 4, or both.
Answer: 2/3
3- MARKS QUESTIONS ( 5 questions)
1-. Suppose you have two coins which appear identical in your pocket. You known that one is fair and one is
two headed. If you take one out, toss it and get a head, what is the probability that it was a fair coin ?
Answer : 1/3
2-. Three bags contains a number of red and white balls as follow:
Bag I : 3 red balls; Bag II : 2 red balls; bag III : 3 white balls
The probability that bag I will be chosen and a ball is selected from it is i/6, I = 1,2,3. If a white ball is
selected, what is the probability that it come from Bag III ?
Answer : 9/11
3-. A shopkeeper sells three types of seeds A-1, A-2 and A-3. They are sold as a mixer where the proportions
are 4 : 4 : 2 respectively. The germination rates of three types of seeds are 45%, 60% and 35% respectively.
Calculate the probability that it will nor germinate given that the seed is of type A-3.
Answer: 35/100
99

4-. There are three coins. One is two headed coin, another is based coin that comes up heads 75% of the times
and third is an unbased coin. One of the three coins is choosen at random and tossed, If shows head, what is
the probability that it was the two headed coin ?
Answer: 4/9
5-. Bag A contain 3 red and 5 black balls, white bag B contains 4 red balls and 4 black balls. Two balls are
transferred at random from bag A to bag B and then a ball is drawn from bag B at random. If the ball drawn
from bag B is found to be red, find the probability that two red balls where transferred from bag A to bag B.

5- MARKS QUESTIONS ( 2 questions)


1-. Consider the experiment of tossing a coin. If the coin shows head, toss it again but if it shows tail, then
throw a die. Find the conditional probability of the event that ‘the die shows a number greater than 4’ given
that ‘there is at least one tail’. (Answer: 2/9)
2-. In a factory which manufactures bolts, machines A, B and C manufacture respectively 25%, 35% and 40%
of the bolts. Of their outputs, 5, 4 and 2 percent are respectively defective bolts. A bolt is drawn at random
from the product and is found to be defective. What is the probability that it is manufactured by the machine
B? ( Answer : 28/69)

CASE STUDY BASED QUESTIONS (PROBABILITIES )


CASE STUDY - 1
Attempt any four sub-parts from the questions given. Each sub-part carries 1 mark.
I Read the following text and answer the following
questions basis of the same:
A coach is trying 3 players. He observed that the
player A can hit the target 4 times in 5 shots, the
player B can hit the target 3 times in 4 shots and
the player C can hit the target 2 times in 3 shots.

1. Let the target is hit by A, B: the target is hit by B and C: the target is hit by A and C.
Then, the probability that A, B and, C all will hit, is:
(A) 4/5 (B) 3/5 (C) 2/5 (D) 1/5 [Correct answer is C]
2. What is the probability that B, C will hit and A will lose?
(A) 1/10 (B) 3/10 (C) 7/10 (D) 4/10 [Correct answer is A]
3. What is the probability that ‘any two of A, B and C will hit?
(A) 1/30 (B) 11/30 (C) 17/30 (D) 13/30 [ Correct answer is D ]
4. What is the probability that ‘none of them will hit the target’?
(A) 1/30 (B) 1/60 (C) 1/15 (D) 2/15 [Correct answer is B ]
5. What is the probability that at least one of A, B or C will hit the target ?
6. (A) 59/60 (B) 2/5 (C) 3/5 (D) 1/60 [Correct answer is A ]

CASE STUDY - 2
100

II. Read the following text and answer the following questions on the basis of the same:
The reliability of a COVID PCR test is specified as follows:
Of people having COVID, 90% of the test detects the
disease but 10% goes undetecte. Of people free of COVID,
99% of the test is judged COVID negative but 1% are
diagnosed as showing COVID positive. From a large
population of which only 0.1% have COVID, one person is
selected at random, given the COVID PCR test, and the
pathologist reports him/
her as COVID positive.

Q. 1. What is the probability of the ‘person to be tested


as COVID positive’ given that ‘he is actually having COVID’?
(A) 0.001 (B) 0.1 (C) 0.8 (D) 0.9 [Ans. Option (D) is correct. ]
Q. 2. What is the probability of the ‘person to be tested
as COVID positive’ given that ‘he is actually not having COVID’?
(A) 0.01 (B) 0.99 (C) 0.1 D) 0.001 [Ans. Option (A) is correct. ]
Q. 3. What is the probability that the ‘person is actually
not having COVID’?
(A) 0.998 (B) 0.999 (C) 0.001 (D) 0.111 [Ans. Option (B) is correct. ]
Q. 4. What is the probability that the ‘person is actually
having COVID given that ‘he is tested as COVID positive’?
(A) 0.83 (B) 0.0803 (C) 0.083 (D) 0.089 [Ans. Option (C) is correct ].
Q.5. What is the probability that the ‘person selected will be diagnosed as COVID positive’?
(A) 0.1089 (B) 0.01089 (C) 0.0189 (D) 0.189 [Ans. Option (B) is correct.]
CASE STUDY - 3
III. Read the following text and answer the following
questions on the basis of the same:
In answering a question on a multiple choice test for class
XII, a student either knows the answer or guesses. Let 3/5
be the probability that he knows the answer and 2/5 be the
probability that he guesses. Assume that a student who
guesses at the answer will be correct with probability 1/3.
Let E1, E2, E be the events that the student knows the
answer, guesses the answer and answers correctly
respectively.

Q. 1. What is the value of P(E1) ?


(A) 2 /5 (B) 1/ 3 (C) 1 (D) 3/5 [ Ans. Option (D) is correct].
Q. 2. Value of P(E/ E1,) is:
A) 1/3 B) 1 (C) 2/3 (D) 4/5 [Ans. Option (B) is correct.]
Q.3. ∑𝑘=2𝑘=1 P( E/Ek) P(Ek) Equals :
101

(A) 11/5 (B) 4/15 (C) 1/5 (D) 1 [ Ans. Option (A) is correct.]
𝑘=2
Q.4. ∑𝑘=1 P( Ek) Equals :
(A) 1/3 (B) 1/5 (C) 1 (D) 3/5 [ Ans. Option (C) is correct.]
Q.5 What is the probability that the student knows the answer given that he answered it
correctly?
(A) 2/11 (B) 5/3 (C) 9/11 (D) 13/3 [ Ans. Option (C) is correct.]
KENDRIYA VIDYALAYA SANGATHAN JABALPUR REGION
BLUE PRINT
CLASS XI: SAMPLE PAPER 2023-24
SUBJECT: MATHEMATICS (041)

S.NO UNITS NAME OF SECTION –A SECTION – B SECTION – C SECTION – D SECTION – E TOTAL


CHAPTERS (01 MARK (VSA) (SA) (LA) (CBQ)
TOPIC EACH) (02 MARKS (03 MARKS EACH) (05MARKS EACH) (04 MARKS EACH)
MCQ ARQ EACH)

1. SETS 1(1) 1(1) 4(1) 6(3)


2. SETS AND RELATIONS AND 2(2) 6(2) 8(4)
FUNCTIONS FUNCTIONS
(23) TRIGONOMETRIC 1(1) 3(1) 5(1) 9(3)
3.
FUNCTIONS
4. COMPLEX 1(1) 3(1) 4(2)
NUMBERS AND
QUADRATIC
EQUATIONS
5. LINEAR 5(1) 5(1)
INEQUALITIES
6. PERMUTAIONS 1(1) 1(1) 4(1) 6(3)
ALGEBRA (25)
AND
COMBINATIONS
7. BINOMIAL 2(2) 2(1) 4(3)
THEOREM

8. SEQUENCE AND 1(1) 2(1) 3(1) 6(3)


SERIES

9. CO ORDINATE STRAIGHT LINES 1(1) 5(1) 6(2)


GEOMETRY
10. (12) CONIC SECTION 1(1) 2(1) 3(2)
11. 3D GEOMETRY 1(1) 2(1) 3(2)
12 CALCULAS LIMITS AND 3(3) 2(1) 3(1) 8(5)
(08) DERIVATIVES

13 STATISTICS STATISTICS 1(1) 5(1) 6(2)


AND
14 PROBABILITY PROBABILITY 2(2) 4(1) 6(3)
(12)
TOTAL 18(18) 2(2) 10(5) 18(6) 20 (4) 12(3) 80(38)
KENDRIYA VIDYALAYA SANGATHAN
JABALPUR REGION
SAMPLE PAPER 2023-24
CLASS - XI

MATHEMATICS (041)

________________________________________________________________________________

Time allowed: 3 Hours Maximum Marks: 80


___________________________________________________________________________________________________________

General Instructions:

This Question paper contains - five sections A, B, C, D and E. Each section is compulsory.
i However, there are internal choices in some questions. You have to attempt only one of the
alternatives in all such questions.

ii Section A has 18 MCQ’s and 02 Assertion - Reason based questions of 1 mark each.

iii Section B has 5 Very Short Answer (VSA) - type questions of 2 marks each.

iv Section C has 6 Short Answer (SA) - type questions of 3 marks each.

v Section D has 4 Long Answer (LA) - type questions of 5 marks each.

vi Section E has 3 source based / case based / passage based / integrated units of
assessment (4 marks each) with sub parts.

vii Use of calculator is not permitted. You may ask for logarithmic tables, if required.

Page 1 of 11
SECTION – A

Direction (Q.1 - Q.18) - There are multiple choice type questions. Choose the correct
answer:

Q. QUESTIONS MARKS
NO.

1 "The complement of the intersection of two sets is the union of their complements." 1
This statement is called:

(a) Complement Law (b) Associative Law


(c) Idempotent Law (d) De Morgan's Law

2 For the Venn - diagram given below, the set ( ) ( ) is: 1

(a) {(3,4), (3,7), (3,9), (8,4), (8,7), (8,9)} (b) {(4,8), (9,8), (7,8), (4,3), (9,3), (7,3)}

(c) {(8,4), (8,9), (8,7), (4,3), (9,3), (7,3)} (d) {(4,8), (9,8), (7,8), (3,4), (3,9), (3,7)}

3 If is the identity function and is the modulus function then which of the following 1
is true?

(a) ( )( ) { (b) ( )( ) {

(c) ( )( ) { (d) ( )( ) {

4 The angle made by an arc of unit length of a unit circle is 1


(a) 1 degree (b) 360 degree

(c) 1 radian (d) radians

5 How many chords can be drawn through 15 points on a circle? 1


(a) 30 (b) 90
(c) 105 (d) 210

Page 2 of 11
Q. QUESTIONS MARKS
NO.

6 In the figure given below, the point ( ) represents the complex number 1
. The length of line segment is called:

(a) Conjugate of .

(b) Modulus of .

(c) Real part of .

(d) Imaginary part of .

7 Consider the following statements about Pascal’s triangle: 1

(i) Every number in a row is the sum of the two numbers diagonally above it
(ii) Sum of the numbers in a row is equal to 2n
(iii) It is a triangular arrangement of numbers that gives the coefficients in the
expansion of any binomial expression, such as (x + y)n

Which of the (i), (ii) and (iii) are always true:

(a) Only (i) and (ii) (b) Only (i) and (iii)
(c) Only (ii) and (iii) (d) All are true

8 Total number of terms in the expansion of ( ) ( ) after simplification 1


are:
(a) 26 (b) 52
(c) 25 (d) 104

9 If the Arithmetic Mean and Geometric Mean of two different positive real 1
numbers and are A.M. and G.M. respectively, then which of the following
is always true?

(a) A.M. G.M. (b) A.M. G.M.


(c) A.M. G.M. (d) A.M. G.M.

Page 3 of 11
Q. QUESTIONS MARKS
NO.

10 The equation of straight line parallel to the at a distance 5 units below it is: 1

(a) (b)

(c) (d)

11 The coordinates of the centre and radius of the circle whose equation in standard 1
form is ( ) ( ) , are:

(a) Centre: ( ), Radius: 4 (b) Centre: ( ), Radius: 4

(c) Centre: ( ), Radius: 16 (d) Centre: ( ), Radius: 16

12 In the figure given below, the names of the planes 1, 2 and 3 are respectively: 1

Plane - 1 Plane - 2 Plane - 3

(a) YZ ZX XY

(b) XY YZ ZX

(c) XZ YZ XY

(d) XY ZX YZ

13 The value of is: 1

(a) 0 (b) 1
(c) (d) Does not exist

14 Derivative of a function ( ) with respect to at is: 1

(a) Slope of line parallel to at the point ( ( ))

(b) Slope of line parallel to at the point ( ( ))

(c) Slope of tangent to the curve ( ) at the point ( ( ))

(d) Slope of the line passing through origin and the point ( ( ))

Page 4 of 11
Q. QUESTIONS MARKS
NO.

15 The value of ( ) is: 1

(a) (b)

(c) (d)

16 Which of the following is not a measure of dispersion: 1

(a) Median (b) Range

(c) Mean deviation (d) Standard deviation

17 If A and B are two mutually exclusive events then which of the following are true: 1
(i)
(ii) ( ) ( )
(iii) ( ) ( )
(iv) ( ) ( ) ( )

(a) Only (i) and (ii) (b) Only (ii) and (iii)
(c) Only (i) and (iv) (d) Only (iii) and (iv)

18 Three integers are chosen at random from 1 to 10. The probability that they are 1
non-consecutive is:

(a) (b)

(c) (d)

ASSE RTION - REASON BASED QUESTIONS

Direction (Q.19 - Q.20) -


In the following questions, a statement of Assertion (A) is followed by a statement of Reason (R).
Choose the correct answer out of the following choices:

(a) Both A and R are true and R is the correct explanation of A.


(b) Both A and R are true but R is not the correct explanation of A.
(c) A is true but R is false.
(d) A is false but R is true.

Page 5 of 11
Q. QUESTIONS MARKS
NO.

19 Assertion (A): 1
If for any two sets and , then adjacent
Venn diagram represents correct relationship
between and .

Reason (R): If , then all elements of are also in .

20 Assertion (A): 1
In the given figure total number of rectangles of
any size is 150.

Reason (R): In a grid ( horizontal and vertical straight lines) total

numbers of rectangles formed is (Selecting two

horizontal and two vertical straight lines simultaneously)

SECTION – B

Direction (Q.21 - Q.25) - This section comprises of very short answer type-questions (VSA)
of 2 marks each.

Q. QUESTIONS MARKS
NO.

2
21 Using binomial theorem, expand ( )
OR
Using the binomial theorem, show that always leaves remainder 1 when
divided by 25.

22 Insert two arithmetic means between 49 and 28. 2

Page 6 of 11
Q. QUESTIONS MARKS
NO.

23 Find the equation of parabola with vertex ( ) and focus ( ). 2


OR
Find the eccentricity and the coordinates of foci of the ellipse .

24 Find the coordinates of a point on y-axis which are at a distance of √ from the 2
point P (3, –2, 5).

25 Evaluate: If ( ) ( ) and , then find . 2

SECTION – C

Direction (Q.26 - Q.31) - This section comprises of short answer type-questions (SA) of 3
marks each.

Q. QUESTIONS MARKS
NO.

26 Given a relation *( ) +, where is the set of all 3


whole numbers. Find:

(i) in roster form (ii) Domain of (iii) Range of

27 Find the domain and range of the function: ( ) 3

OR

The function is defined by ( ) { . Draw the graph of ( ).

28 Prove that: 3

29 Reduce ( )( ) to the standard form. 3

30 Find the sum of the sequence: to terms. 3


OR
In a G.P, the 3rd term is 24 and 6th term is 192. Find the 10th term.

Page 7 of 11
Q. QUESTIONS MARKS
NO.

31 Find the derivative with respect to : 3

OR

SECTION – D

Direction (Q.32 - Q.35) - This section comprises of long answer type-questions (LA) of 5
marks each.

Q. QUESTIONS MARKS
NO.

32 If , lies in 3rd Quadrant, then find the values of (i) (ii) 5

OR

Prove that:

33 A solution of 8% boric acid is to be diluted by adding a 2% boric acid solution to it. 5


The resulting mixture is to be more than 4% but less than 6% boric acid. If we have
640 litres of the 8% solution, how many litres of the 2% solution will have to be
added?

34 Solve the questions (i), (ii) and (iii) given below:

(i) Find the equation of the line which intersects at a distance 2 units in 2

the right of origin and at distance 3 units below to the origin.


(ii) Find the distance of point ( ) from the line . 1

(iii) Find the angle between the and the line joining the points ( ) and 2
( ).
OR
Find equations of lines which pass through the point ( ) and the sum of its 5
intercepts on the axes is 14.

35 Calculate mean, variance and standard deviation for the following distribution. 5

Classes 30 - 40 40 - 50 50 - 60 60 - 70 70 - 80 80 - 90 90 - 100

Frequency 3 7 12 15 8 3 2

Page 8 of 11
SECTION – E

Direction (Q.36 - Q.38) - This section comprises of 3 case-study / passage-based questions


of 4 marks each.

Q. QUESTIONS MARKS
NO.
36 Case-Study: Read the Case study given below and attempt (i), (ii) and any 4
one subparts of (iii):
Ravi, a student of class XI had been anxiously waiting for having a get together
with his classmates as he could not be with them for a stretch of two years in
Corona Period. He invited four of his best friends Amar, David, sheetal and Fatima
to his birthday party. After cutting cake they gathered to take a group photograph of
all of them sitting in a row.

(i) How many different photographs can be clicked? 1


(ii) In how many of these photographs would Fatima be not sitting at middle? 1
(iii)In how many of these photographs would Ravi and Amar be sitting side by side? 2
OR (The option is only for part (iii))
In how many of these photographs would David and Sheet be not sitting
together?

37 Case-Study: Read the Case study given below and attempt (i), (ii) and any
one subparts of (iii): 4
Four persons Abdul , Kavita, Monika and Pramod are appearing in an interview to
get a job in a software company. Chances of Abdul’s selection are same as that of
Pramod. Chances of Kavita’s selection are double that of Abdul. Chances of
Monika’s selection are four times that of Abdul. One and only one among these are
sure to be selected.
Page 9 of 11
Q. QUESTIONS MARKS
NO.

(i) What is the probability that Abdul gets selection? 1


(ii) What is the probability that Kavita gets selection? 1
(iii) What is the probability that Abdul or Monika get selection? 2

OR (The option is only for part (iii))

What is the probability that Pramod does not get selection?

38 Passage-based question: Study the passage and table given below


and answer the questions (i) and (ii) given below: 4

The intervals are defined as the set of all real numbers lying between two given real
numbers (end points / boundary points). It is a way of writing subsets of the set of
all real numbers. Based on the inclusion / exclusion of end points the intervals are
classified as – closed, open and semi closed / semi open intervals as shown in the
following table.

Page 10 of 11
Q. QUESTIONS MARKS
NO.

Intervals are sets so we can combine two or more intervals using set operations.

(i) To join the Indian Army under technical entry scheme the age of a candidate 2
must be more than 16½ years and not above 19½ years. Represent the age
limit using the interval.

(ii) According to weather report of Meteorological department the hottest month in 2


Srinagar is July (minimum temperature 6°C, maximum temperature 32°C) and
the coldest are December – January (temperature is between −15°C and 0°C).
Represent the range of temperature in both the seasons as a single interval
using the set operations.

Page 11 of 11
KENDRIYA VIDYALAYA SANGATHAN, JABALPUR REGION
SAMPLE PAPER 2023-24
MARKING SCHEME
MATHEMATICS (041) CLASS: XI
SECTION - A
Q.NO. ANSWER SOLUTION MARKS
1 d De Morgan's Law 1

2 a {(3,4), (3,7), (3,9), (8,4), (8,7), (8,9)} 1

3 b ( )( ) 2 1

( )( ) ( ) ( ) | |
( )( ) ( ) ( )
( )( ) ( ) ( )

4 c 1 radian 1
5 c 1

6 b Modulus of 1
7 d All are true 1
8 a 26 1
●( ) has 52 terms all are positive
●( ) has 52 terms - 26 positive and 26 negative
● 26 negative terms of ( ) cancel with the 26
corresponding positive terms of ( )
● 26 positive terms of ( ) combine with 26
corresponding positive terms of ( )
9 d A.M. G.M. 1
10 a 1
11 a Centre: ( ), Radius: 4 1

12 b Plane - 1 - XY Plane - 2 - YZ Plane - 3 - ZX 1


13 b 1( ) 1

14 c Slope of tangent to the curve ( ) at the point ( ( )) 1

15 d . . / / 1

16 a Median 1

17 c Only (i) and (iv) 1

18 b ● Total number of selections 1


● Cases for consecutive integers are ( )( )
( )( )( )( )( )( )
● Total number of Cases for consecutive integers = 8
● Total number of favourable cases = 120 - 8 = 112
● Required probability

Page 1 of 9
Q.NO. ANSWER SOLUTION MARKS
19 d A is false but R is true 1
20 a Both A and R are true and R is the correct explanation of A 1

SECTION - B
Q.NO. SOLUTION MARKS

21  ( ) . / ( ) . / ( ) . / ( ) . / 1

( ) . / (Applying binomial theorem)

 (Simplifying) 1

OR
 ( ) ( ) 1

 . / 1

22  Let the two A.M. are and are in A.P. ½

 ½
 ½
 ½

23  It is downward parabola of the form with 1

 Hence equation of parabola is: 1

OR
 Equation of ellipse in standard form:

(It is vertical ellipse in standard form: ) ½

 , √ √ ½


½
 Foci: ( ) i.e. ( √ ) ½

24  A point on y-axis is ( ) ½

 Writing and applying distance formula 1

 Getting and writing answer ( ) and ( ) ½

Page 2 of 9
Q.NO. SOLUTION MARKS

25  . / 1

 ( ) ½

 ½

SECTION - C

Q.NO. SOLUTION MARKS

26  *( )( )( )( )+ 1
 Domain: * +
1
 Range: * + 1

27  Finding domain: 2 3 1

 Separating 1

 Finding range: 2 3 1
OR
 Drawing the graph of: ( ) 1

 Drawing the graph of: ( ) 1

 Drawing the graph of: ( ) 1

Page 3 of 9
Q.NO. SOLUTION MARKS

28  ( ) 1

 1

 1
√ √

29  . /. / ( )(
. / ½
)

 1

 1
( ) ( )

( )
 ½

30  to terms
 ( to terms) ½
 ( to terms) ½
 ,( ) ( ) ( ) ( ) - ½
 ,( ) - ½
( ) ( )
 0 1 0 1 1

OR

 ½

 ½

 ½

 Getting ½

 Getting 1

Page 4 of 9
Q.NO. SOLUTION MARKS

31 

( ) ( ) ( )
 1
( )
…….. (1)
(Applying quotient rule)

 Calculation of ( ) (Applying product rule) ½

 Calculation of ( ) ½

 Substituting in (1) and simplifying to get


1
( )

REMARK: It can also be solved as follow:

 and proceed as above to get


( )

OR

( ) ( )
 ……. (i) 1

(Applying quotient rule)

 Calculation of ( ) ½

 Calculation of
½
 Substituting in (1) and simplifying to get

1

Page 5 of 9
SECTION - D
Q.NO. SOLUTION MARKS

32  (As given) 1
(i)
 1


 . / √ 1


 . /

(ii)
 1


 √
1

 . /
OR

 ½

 ( ) ½

 ( ) 1

 . / ½

 ( ) ½

 ( ) ½

 ( )
½

 . /
½
 √ ½
33  Let litres of 2% solution be added then total mixture = litres ½
 Formation of inequality: 2% of 8% 640 4% of ( )
1

1
 Formation of inequality: 2% of 8% 640 6% of ( ) 1

1
Hence the amount of 2% solution to should be more than 320 litres
½
but less than 1280 litres.
Page 6 of 9
34 (i)
 intercept: ½
intercept:

 Equation of line: 1

 (Simplifying) ½
(ii)
( ) ( )
 Distance | | units 1
√ ( )

(iii)
 Slope of ½
 Slope of line joining the points ( ) and ( )
½
 Calculation of Angle 1
OR
 intercept:
1
intercept:

 Equation of line: 1

 Line is passing through ( )


1
 Simplifying as
 Solving above equation: 1
 Getting equations of lines: 1

35 Frequency Mid - Point ̅ For


Classes ( ) correct
( )
mean - 2
30 - 40 3 35 105 -27 729 2187
Marks
40 - 50 7 45 315 -17 289 2023

50 - 60 12 55 660 -7 49 588

60 - 70 15 65 975 3 9 135
For
70 - 80 8 75 600 13 169 1352 correct
variance
80 - 90 3 85 255 23 529 1587
-2
90 - 100 2 95 190 33 1089 2178 Marks

TOTAL N = 50 3100 10050

For
 Calculation of Mean: ̅ ∑
correct
S.D. - 1
 Calculation of Variance: ∑ Marks
 Calculation of S.D.: √

Page 7 of 9
SECTION - E

Q.NO. SOLUTION MARKS

36 (i) 1

(ii)
 When Fatima is sitting at middle, the numbers of photographs are:
1

 When Fatima is not sitting at middle, the numbers of photographs are:

(iii)
 When Ravi and Amar be sitting side by side, the numbers of
2
photographs are:
OR
 When David and sheetal be sitting together, the numbers of 1

photographs are:

 When David and sheetal be not sitting together, the numbers of 1


photographs are:

37 (i) Consider the events 1


Abdul gets selection
Kavita gets selection
Monika gets selection
Pramod gets selection
( ) ( ) ( ) ( )
( ) ( ) ( ) ( )

( )

(ii) ( ) ( ) 1

(iii) ( ) ( ) ( ) (A and C are Mutually exclusive) 2


OR

( ) ( ) ( )

Page 8 of 9
Q.NO. SOLUTION MARKS

38 (i) 2

The open - closed interval: ( -

(ii) , - ( ) 2

Page 9 of 9

You might also like